Сохранен 534
https://2ch.hk/sci/res/335184.html
24 декабря Архивач восстановлен после серьёзной аварии. К сожалению, значительная часть сохранённых изображений и видео была потеряна. Подробности случившегося. Мы призываем всех неравнодушных помочь нам с восстановлением утраченного контента!

Математика, тред 42

 Аноним 02/02/16 Втр 02:42:41 #1 №335184 
14543701613030.png
Этот тред будет посвящён малоизвестным (среди любителей) фактам из теории множеств. Я начну.

Есть теорема: всякое бесконечное множество содержит счётное подмножество. Любительское доказательство этой теоремы (встречающееся, однако, во многих учебниках) таково. Пусть M - бесконечное множество. Тогда оно непусто, то есть содержит элемент i1. Так как M бесконечно, множество M\{ i1 } тоже бесконечно и, стало быть, содержит элемент i2 (значком \ обозначена теоретико-множественная разность). Множество M\{ i1, i2 } тоже бесконечно и, стало быть, содержит элемент i3. Продолжая этот процесс, получаем, что M содержит счётное множество {i1, i2, i3, ... }.

Малоизвестный нюанс здесь в том, что мы, вообще-то, не доказали, что совокупность {i1, i2, i3, ... } является множеством. Чтобы доказать это, нужно воспользоваться одной из форм аксиомы выбора. Без аксиомы выбора, оказывается, нельзя доказать, что во всяком бесконечном множестве есть счётные подмножества. Коэн доказал, что с системой аксиом Цермело-Френкеля без аксиомы выбора совместно утверждение о существовании бесконечных множеств, не содержащих счётных подмножеств.

Некоторые люди ведут войну с аксиомой выбора, утверждая, что её следствия парадоксальны. Но отказ от аксиомы выбора уничтожает гораздо больше теорем, чем думают эти люди.

Предыдущий: >>331617 (OP)
Архивы Аноним 02/02/16 Втр 02:43:30 #2 №335185 
1. https://arhivach.org/thread/18638/
2. https://arhivach.org/thread/27246/
3. https://arhivach.org/thread/27696/
4. https://arhivach.org/thread/38709/
5. https://arhivach.org/thread/46502/
6. https://arhivach.org/thread/48852/
7. https://arhivach.org/thread/52165/
8. https://arhivach.org/thread/56479/
9. https://arhivach.org/thread/63306/
10. https://arhivach.org/thread/70618/
11. https://arhivach.org/thread/74342/
12. https://arhivach.org/thread/74341/
13v1. https://arhivach.org/thread/76561/
13v2. https://arhivach.org/thread/92428/
14. https://arhivach.org/thread/78408/
15. https://arhivach.org/thread/79152/
16. https://arhivach.org/thread/82499/
17. https://arhivach.org/thread/92427/
18. https://arhivach.org/thread/84722/
19. https://arhivach.org/thread/87923/
20. https://arhivach.org/thread/91329/
21. http://arhivach.org/thread/93067/
22. https://arhivach.org/thread/94240/
23. https://arhivach.org/thread/95680/
24. https://arhivach.org/thread/96720/
25. https://arhivach.org/thread/99481/
26. https://arhivach.org/thread/100880/
27. https://arhivach.org/thread/101335/
28. http://arhivach.org/thread/106743/
29. https://arhivach.org/thread/109198/
30. https://arhivach.org/thread/114111/
31. https://arhivach.org/thread/116099/
32. https://arhivach.org/thread/118093/
33v1. https://arhivach.org/thread/122613/
33v2. https://arhivach.org/thread/122615/
34. https://arhivach.org/thread/123717/
35. https://arhivach.org/thread/128822/
36. https://arhivach.org/thread/129039/
37. https://arhivach.org/thread/131462/
38. https://arhivach.org/thread/138362/
39. https://arhivach.org/thread/138429
40. http://arhivach.org/thread/140404/
41. https://arhivach.org/thread/142386/
Аноним 02/02/16 Втр 04:22:22 #3 №335199 
>>335184 (OP)
У ощущение лютой хуйни от этого.
Во-первый, от того, что надо доказывать, что бесконечное множество содержи счетное. Это же очевидно, зачем это доказывать?

И второе вообще пушка:
>>мы, вообще-то, не доказали, что совокупность {i1, i2, i3, ... }
Ну вот же оно: i1, i2, i3... Счетное множество. Бери и считай. В чём, блядь, проблема?
Аноним 02/02/16 Втр 04:36:51 #4 №335200 
>>335199
>Это же очевидно
Нет, это не очевидно. Здесь утверждается, что алеф-нуль - наименьший из кардиналов, грубо говоря. Но почему это должно быть так? Легко можно представить кардиналы, которые меньше алеф-нулевого.

>В чём, блядь, проблема?
Аксиомы теории множеств были введены как раз для того, чтобы отличить те совокупности сущностей, которые являются множествами, от тех совокупностей сущностей, которые не являются множествами. Например, совокупность всех множеств не является множеством, но совокупность всех подмножеств некоего данного множества является множеством. ZF без аксиомы выбора не позволяют доказать, что вот эта вот совокупность {i1, i2, i3, ... } является множеством, а ZF с аксиомой выбора - позволяют.
knuebok 02/02/16 Втр 04:54:15 #5 №335204 
> Легко можно представить кардиналы, которые меньше алеф-нулевого.
И какие же это?

Строго говоря, дело тут не в том, чтобы "доказать, что совокупность {i1,i2,i3,...}" является множеством, ведь ZFC вообще не имеет дело ни с чем, кроме множеств, поэтому в ней не формализуется задача "доказать, что заданная совокупность - это множество", а дело в том, что вот тут:
>Продолжая этот процесс, получаем, что M содержит счётное множество {i1, i2, i3, ... }.
мы как бы "проделываем бесконечное рассуждение", которые в математике не допускаются. В классической, по крайней мере. С помощью аксиомы выбора можно проделать конечное рассуждение, которое докажет существование счётного подмножества, но для этого нужно для начала строго определится с определением бесконечного множества.
Аноним 02/02/16 Втр 05:18:26 #6 №335206 
14543795062940.png
>>335204
>ведь ZFC вообще не имеет дело ни с чем
Я придерживаюсь идеи, что аксиомы вводятся лишь с целью уточнить уже имеющиеся построения. Умозрительные, если угодно. Если система аксиом адекватна имеющимся интуитивным, "доаксиоматическим" построениям, то это хорошая, годная система аксиом. Если же аксиомы не позволяют доказать интуитивно очевидные утверждения, то следует искать другие, более адекватные аксиомы. Идея совокупности, к которой я обращаюсь, - это доаксиоматическая, фантастическая идея. Не какой-то объект ZFC. Моя мысль в том, что ZF без аксиомы выбора не адекватна наивной теории множеств, потому что не позволяет доказать, что некая совокупность, которая очевидно должна быть множеством, действительно им является.

>мы как бы "проделываем бесконечное рассуждение", которые в математике не допускаются
Бесконечное рассуждение можно инкапсулировать в какую-то конечную аксиому же, и таким образом спокойно использовать.

>И какие же это?
Это кардиналы бесконечных множеств, которые меньше счётного. Тоже фантастический объект, я не знаю, изучал ли кто-нибудь такие штуки.

Конкретно теорему Коэна я позаимствовал у Вавилова. Вавилов, в свою очередь, ссылается на книжку Йеха "Теория множеств и метод форсинга".
knuebok 02/02/16 Втр 05:36:53 #7 №335207 
Теорема Коэна - результат известный, но он ничего не утверждает о существовании кардиналов меньше алеф нуль.

Мысль я понял, и позиция эта известная и понятная. Но проблема аксиомы выбора не в том, что она доказывает контринтуитивные результаты, а в том, что доказательства через АС неконструктивны. Мы, конечно, можем разрезать шар на два шара, но если нам дадут конкретную координату точки, то мы не сможем сказать - будет ли она после разрезания принадлежать первому шару, или второму. Более того, мы даже не можем сказать, сможем ли мы вообще когда-нибудь узнать это или нет, то есть с точки зрения конструктивиста - у нас ровно 0 полезной информации от этой аксиомы. Но с позиций некоторой вселенской истины мне такой взгляд тоже кажется недальновидным: мне очевидно, что должны существовать объекты, которые человечество не сможет сконструировать никаким образом, но которые всё же существуют и своим существованием делают всю эту вашу математическую картину эстетичнее, что ли.
Платиновая околофилософия на sci!
Аноним 02/02/16 Втр 10:45:07 #8 №335259 
Ебать 42 тред. Был тут только в первом.
Аноним 02/02/16 Втр 11:05:35 #9 №335263 
>>335259
Выебал тебя еще в первом треде.
Аноним 02/02/16 Втр 12:33:37 #10 №335272 
У мен еще вопрос по касательным. Если надо порвести касательную к функции со степенью больше квадрата, как тогда производную брать? Например касательную к функции x^3 как провести? Ведь если взять производную один раз, она будет в кквадрате и прямой не получится. Что делать, отбрасывать степень или брать производную пока степень не исчезнет?
Аноним 02/02/16 Втр 12:51:45 #11 №335274 
>>335272
1. Производную бери один раз.
2. В получившееся выражение подставляй значение икса точки касания.

Если вам помогло это сообщение, ответьте "ЗЕЛЁНЫЙ СЕРХРАЗУМ БУБУГИ".
Аноним 02/02/16 Втр 12:54:05 #12 №335276 
Посаны, я ньюфаг. Верно ли, что каждый раз, когда мы строим счетную последовательность, мы неявно используем аксиому выбора? И каким образом мы ее используем?
Аноним 02/02/16 Втр 12:56:30 #13 №335277 
Гайз, ну поясните мне за классы смежности, нормальные подгруппы и факторгруппы, плиз. Я туповат и не понимаю смысла введения в алгебре конструкций типа aba^-1. И что означает если эта конструкция принадлежит какой-либо подгруппе (это про нормальные группы)?
Аноним 02/02/16 Втр 12:57:47 #14 №335278 
>>335184 (OP)
Кстати, это утверждение следует из того, что если существует сюръекция из A в B, то card(A) >= card(B). Вот пост на mse про это:
http://math.stackexchange.com/questions/726611/surjective-map-and-cardinality
Аноним 02/02/16 Втр 12:57:52 #15 №335279 
>>335274
Точно, я и забыл что надо подставлять значение икса в точке. Спасибо.
Аноним 02/02/16 Втр 12:58:24 #16 №335280 
2+2= ?
Аноним 02/02/16 Втр 12:59:02 #17 №335281 
>>335280
(+ 2 2)
Аноним 02/02/16 Втр 12:59:33 #18 №335282 
>>335200
А разве множества появляются не после их определения?

И из определения уже будет ясно, можно там что-то посчитать или нет, можно выделить какие-то элементы или нет.

Например, взяли множество всех множеств, поняли, что это парадоксальная ерунда, и не рассматриваем его.

Рассматриваем рациональные числа или поле каких-нибудь многочленов и понимаем, что это счетные множества. Или счетность рациональных чисел тоже требует для доказательства аксиому выбора и прочие умные слова (смысл которых я не очень понимаю)?
Аноним 02/02/16 Втр 13:03:22 #19 №335283 
>>335281
Да.
Аноним 02/02/16 Втр 13:06:26 #20 №335285 
>>335184 (OP)
Пикрелейтед
http://math.stackexchange.com/questions/55068/can-you-raise-a-number-to-an-irrational-exponent
Аноним 02/02/16 Втр 13:19:55 #21 №335287 
>>335200
>Это кардиналы бесконечных множеств, которые меньше счётного.
Таких нету: в рамках ZFC они существовать не могут.
Аноним 02/02/16 Втр 13:45:55 #22 №335290 
>>335281
Сверхразум вкатился.
Аноним 02/02/16 Втр 13:54:54 #23 №335291 
>>335287
А в рамках ZF?
Аноним 02/02/16 Втр 14:32:57 #24 №335296 
>>335291
Да. ω - наименьший предельный ординал.
knuebok 02/02/16 Втр 15:32:51 #25 №335310 
>Рассматриваем рациональные числа или поле каких-нибудь многочленов и понимаем, что это счетные множества. Или счетность рациональных чисел тоже требует для доказательства аксиому выбора и прочие умные слова (смысл которых я не очень понимаю)?
Не требует.
>А разве множества появляются не после их определения?
Об этом думать так странно. Лучше думать, будто все множества "уже есть", а мы можем только доказывать, что множества с некоторыми свойствами существуют, а с некоторыми - не существуют. И эти доказательства должны быть формально корректными, то есть использовать чётко фиксированные правила вывода из чётко фиксированных аксиом. Например, вот так http://us.metamath.org/mpegif/isinf.html выглядит формально корректное доказательство того, что любое бесконечное множество содержит сколь-угодно большое кон
Аноним 02/02/16 Втр 15:43:55 #26 №335312 
>>335310
Кандл дже
Аноним 02/02/16 Втр 15:51:39 #27 №335316 
>>335310
>все множества "уже есть"
Множество, которое существует в одной системе аксиом, легко может не существовать в другой. Это портит всю метафизику.
Аноним 02/02/16 Втр 16:41:41 #28 №335322 
Вы все заебали меня.
Аноним 02/02/16 Втр 17:10:05 #29 №335327 
>>335310
>>Лучше думать, будто все множества "уже есть"
А что полезного дал этот подход? Какие задачи над привычными нам множествами (разного сорта чисел, например) он позволяет решить?

Мне с позиций дилетанта кажется, что рассуждать о множествах "вообще" и потом ковыряться с доказательствами очевидного это какая-то абстрактная вода.
Аноним 02/02/16 Втр 17:44:56 #30 №335330 
>>335282
Ну ты рассуждаешь как нормальный человек, а они рассуждают так, будто они сверхумные, но на самом деле их "знания" это бред какой-то.

Что вообще доказывает, что множество/ряд чисел/природный феномен/эмоция и т.д. является бесконечным? В нашей Вселенной сейчас ничего бесконечного быть не может, только в вымышленном/мнимом мире могут быть бесконечные процессы. И вообще, бесконечность это не число, его не возможно преобразовать, разделить, помножить, вычесть... Бесконечность это концепция, идея как например справедливость, воля, трусость...
knuebok 02/02/16 Втр 17:48:18 #31 №335332 
>>335316
Ну имеется в виду, что формальная система вместе с моделью фиксирована и относительно неё мы "строим метафизику". В другой формальной системе или даже в той же самой, но с другой моделью - другая метафизика.
>>335327
>А что полезного дал этот подход?
Ну такая философия сейчас более-менее доминирует в математике, то есть неявно предполагается, что любое рассуждение можно при достаточном усердии свести к формальному выводу из аксиом ZFC (или каких-то других). Она, собственно, математику в современном виде и дала. Вообще я не очень понял вопроса, наверное.

>с доказательствами очевидного
Я думал тут весь тред о том, что кому-то не очевидно, что у ZF существуют модели в которых есть бесконечные (= не конечные) множества, несравнимые со счётным, лил.

>>335330
>В нашей Вселенной сейчас ничего бесконечного быть не может
Табу во все поля.
Аноним 02/02/16 Втр 18:06:01 #32 №335333 
>>335332
>неявно предполагается, что любое рассуждение можно при достаточном усердии свести к формальному выводу из аксиом ZFC
А разве нет?
Аноним 02/02/16 Втр 18:06:47 #33 №335334 
>>335332
>Вообще я не очень понял вопроса
Ну вот мы например ввели абстракцию: линейное пространство. С её помощью можно ответить на вопрос, можно ли составить такой многочлен из других многочленов, чтобы он был равен нулю. Или понятие поля: оно позволяет доказать, что нельзя в общем виде решать уравнения степени выше трёх.

А какие задачи помогает решить абстрагирование от конкретных множеств к неким множествам вообще?
knuebok 02/02/16 Втр 18:22:56 #34 №335339 
>оно позволяет доказать, что нельзя в общем виде решать уравнения степени выше трёх.
Выше четырёх.

>А какие задачи помогает решить абстрагирование от конкретных множеств к неким множествам вообще?

Ну если нужны совсем уж рабоче-крестьянские применения: доказательство теоремы Гудстейна, теоремы о гидрах. Очень часто можно доказать существование всяких объектов без их конкретного предъявления тупо сравнив мощности: доказательство существования невычислимых чисел, трансцендентных чисел, неборелевских множеств. Всякие рассуждения по типу "композиция инъективных функций - инъективна" - тоже чисто теоретико-множественные и оторваны от конкретной конструкции.

А вообще, у теории множеств больше фундирующая функция, нежели прикладная: она даёт удобный язык, на котором можно формулировать и высказывать мысли: например конструкция фактормножества по отношению эквивалентности настолько вошла "в кровь" математики, что сложно назвать какой-нибудь раздел, где её бы не было.

Ну и ценность полей, кстати, тоже далеко не в одной теореме Абеля и теории Галуа, я бы сказал, что у полей тоже фундирующая функция, то же определение векторного пространства формулируется в общем случае над произвольным полем, что позволяет сразу построить одну стройную теорию для векторных пространств над Fp,Q,R,C (до определенного момента, по крайней мере).
Аноним 02/02/16 Втр 18:38:08 #35 №335341 
>>335339
>она даёт удобный язык, на котором можно формулировать и высказывать мысли
Похоже физека катиться в эту самую пизду множеств.
knuebok 02/02/16 Втр 18:41:51 #36 №335342 
>>335341
Она скатывается в тех местах, где с теориями уже всё понятно и никто их глобально трогать не будет (классическая механика, СТО), а в местах, которые на стадии формирования (КТП) она не скатывается, просто потому, что там самим физикам ещё не очень понятно что происходит, не говоря уже о какой-то формализации. И это правильно.
Аноним 02/02/16 Втр 18:48:54 #37 №335343 
>>335342
> местах, которые на стадии формирования (КТП) она не скатывается
У меня от тебя МАНЯ-ТЕОРИЯ
Аноним 02/02/16 Втр 19:30:40 #38 №335347 
14544306405620.jpg
>>335277
Грустный бамп котом в миске.
knuebok 02/02/16 Втр 19:53:28 #39 №335348 
>>335347
Ну вот пусть у нас есть группа G и подгруппа H, и мы хотим рассмотреть "группу G с точностью до группы H", то есть писать как-то типа: a=b (mod H). Или, что тоже самое ab^-1 = 1 (mod H). Логично для этих целей попробовать ввести на G отношение эквивалентности a~b <-> ab^-1 лежит в H. Возьмём фактормножество группы G по отношению эквивалентности ~. Будем обозначать класс эквивалентности g как [g]. Очень бы хотелось, чтобы классы эквивалентности тоже можно было перемножать по очевидному правилу (aH)(bH)=[a] x = [ab] = ab H. Если мы посмотрим на равенство aHbH=abH множеств чуть поближе, то мы заметим, что оно выполняется тогда и только тогда, когда bH=Hb, но ведь это и есть требование нормальности группы!
Если коротко: то нормальность группы в точности означает, что мы можем ввести структуру группы на фактормножестве G/H.
Аноним 02/02/16 Втр 20:44:08 #40 №335357 
>>335184 (OP)
Знания по математике - 11 класс.

Нужно хотя бы в общих чертах знать матан. С чего начать? И в какой последовательности изучать? Нужно самые основы.
Аноним 02/02/16 Втр 20:44:56 #41 №335358 
14544350968220.png
14544350968221.gif
>>335207
>Платиновая околофилософия
Потому что все, связанное с бесконечными множествами околофилософия и есть, причем с самого начала. Всем известное "доказательство", основанное на взаимнооднозначном соответствии точек отрезков разной длины (1) ничего не говорит о том, как быть с иными всевозможными вариантами (2), не дающими однозначного соответствия. А следовательно второе всем известное "доказательство", что мощность множества вещественных чисел больше множества рациональных может говорить лишь о том, что найденное Кантором соответствие относится к случаю (2), но это еще никак не говорит о том, что между вещественными и рациональными числами не может существовать взаимнооднозначного соответствия. Кантор доказал лишь, что в показанном им варианте сопоставления нет взаимнооднозначного соответствия, а не то, что такого соответствия не может быть в принципе.
Вся же дальнейшая аксиоматика никакое не развитие, а просто запрещение неудобных случаев. Попытались построить множество всех множеств, получили парадокс - запретить. Ситуация тут ничем в принципе не отличается от античной математики, которая просто запрещала бесконечные величины по той же самой причине - не умела с ними работать.
Ну а уж то, что бесконечного множества в принципе никто никак не смог показать, кроме как в виде буквы, его обозначающей, про это только ленивый тут не кукарекал.
Аноним 02/02/16 Втр 20:51:46 #42 №335362 
>>335357
https://2ch.hk/sci/res/332198.html
Аноним 02/02/16 Втр 20:52:05 #43 №335363 
>>335207
>Теорема Коана
knuebok 02/02/16 Втр 20:58:23 #44 №335368 
>>335358
Неосилятору теоремы Кантора что-то объяснять неинтересно, иди на dxdy.ru, там есть люди которым интересно пережёвывать одно и то же по 170 раз.
Аноним 02/02/16 Втр 21:03:06 #45 №335370 
>>335368
Всегда сам себе задания раздаешь?
knuebok 02/02/16 Втр 21:06:03 #46 №335371 
>>335370
Да я, на самом деле, сам это и написал и сам себе ответил0.
Аноним 02/02/16 Втр 21:33:24 #47 №335380 
Матаноны, чем вы пользуетесь для записи математики в асции? Тех - нечитабельное говно; мне не нужно публиковаться, мне нужно набирать с клавиатуры математику.

Апл не взлетел. Что еще?
Аноним 02/02/16 Втр 21:33:56 #48 №335381 
>>335380
Рисуй в пейнте
Аноним 02/02/16 Втр 21:42:02 #49 №335384 
>>335274
ЗЕЛЁНЫЙ СЕРХРАЗУМ БУБУГИ

Аноним 02/02/16 Втр 21:44:08 #50 №335385 
>>335184 (OP)
Я не понял, какое мы вообще имеем право вводить аксиому выбора, не определив перед этим натуральных чисел?
Аноним 02/02/16 Втр 21:46:52 #51 №335386 
>>335385
Твоя мамка нам лично каждому разрешение дала.
Аноним 02/02/16 Втр 21:54:33 #52 №335390 
>>335368
Посылать найух, на dxdy.ru лишь показывает, что ты как раз сам не понимаешь того о чем говоришь. Если ты такой осиливающий, так будь любезен доказать это своими доводами и аргументами, а не отсылкой на неопределенное множество {X,Y,Й}
knuebok 02/02/16 Втр 21:59:05 #53 №335391 
>>335390
Ну давай попробуем. Для начала ты приводишь полный текст доказательства из какого-нибудь более-менее стандартного источника (если хочешь, то я могу его привести), потом указываешь на ошибку конкретно в этом тексте, а потом уже говорим. Я лично никакого доказательства "... основанного на взаимнооднозначном соответствии точек отрезков разной длины ..." не знаю.
Аноним 02/02/16 Втр 22:02:36 #54 №335393 
>>335380
Что, неужели ни у кого не возникало никогда такой потребности?
Аноним 02/02/16 Втр 22:04:03 #55 №335394 
>>335393
>Предпочитать ASCII, а не TeX.
Нет, ещё не все люди стали конченными говноедами.
Аноним 02/02/16 Втр 22:05:37 #56 №335395 
>>335358
Ну а почему до сих пор человечество не смогло и не научилось работать с бесконечными величинами? Это связано с технологическими недостатками компьютеров, калькуляторов, арифмометров, abacusов или с неправильными основополагающими принципами математики, физики, философии? Вот мы будучи частью космоса и имея разум не можем однозначно сказать бесконечна ли вселенная или она не бесконечная/замкнутая.
Аноним 02/02/16 Втр 22:06:56 #57 №335396 
>>335395
>Ну а почему до сих пор человечество не смогло и не научилось работать с бесконечными величинами?
Да вроде как научилось.
Аноним 02/02/16 Втр 22:07:34 #58 №335397 
>>335394
Неотрендеренный тех читать не очень-то удобно. Он для другого. Я же ясно сказал: делать заметки, а не публиковать.
Аноним 02/02/16 Втр 22:10:47 #59 №335398 
>>335397
Так а ты его рендерь. При достаточной надрочке и скорости печати получается фигарить формулы очень быстро, что для заметок вполне себе.
Аноним 02/02/16 Втр 22:11:17 #60 №335399 
>>335395
>человечество не смогло и не научилось работать с бесконечными величинами?
Лол, как там в 18-м веке?
Аноним 02/02/16 Втр 22:14:19 #61 №335401 
>>335398
Ох, ну не знаю. Я пробовал - неудобно. Когда надо редактировать, приходиться смотреть на отрендеренное, а править неотредеренное, и это... уныло как-то. Да и элементарно много лишних буков.
Аноним 02/02/16 Втр 22:16:22 #62 №335402 
>>335391
Во первых, моя цель не придираться к теоремам и пытаться выискивать соринку в чьем либо глазу, а попытаться понять то, что есть и познал ли я это.

Есть отрезок с двумя точками А и В, на этом отрезке есть точки и количество этих точек, как нас учат еще со школы, бесконечно. Есть другой отрезок с точками С и D, он в 2 раза длиннее отрезка АВ, но между С и D есть точки и количество этих точек тоже бесконечно. Но АВ в 2 раза короче CD. Как это надо понимать?
knuebok 02/02/16 Втр 22:18:05 #63 №335404 
>>335402
Понимать следующим образом: длина отрезка и мощность отрезка как множества точек - две принципиально разные характеристики отрезка, никак между собою не связанные.
Аноним 02/02/16 Втр 22:20:23 #64 №335405 
>>335401
Рисуй. еще раз говорю.
Аноним 02/02/16 Втр 22:23:34 #65 №335406 
>>335405
Я не умею..
Аноним 02/02/16 Втр 22:25:58 #66 №335407 
>>335404
Ты сейчас ответил как поп, навсеволябожья! Мощность отрезка? Это как длина и толщина члена? Так сейчас во всей математике, чтоб объяснить одно ссылаются на другое, выясняешь другое он отсылает на третье, третье на 4-е , а это уже и есть Бесконечное Множество. Что и требовалось доказать.
Аноним 02/02/16 Втр 22:26:51 #67 №335408 
>>335407
Хуйню какую-то дилетантскую сказанул.
knuebok 02/02/16 Втр 22:28:31 #68 №335409 
>>335407
Я из твоего словесного потока ничего не понял, но что-то объяснить тебе пытаться уже не буду.
Аноним 02/02/16 Втр 22:32:22 #69 №335410 
>>335348
>aHbH=abH
А почему не aHbH=abH^2? Мне вот это не понятно. Остальное, вроде бы, складно звучит. Только почему-то у тебя ab^-1, а в учебниках bab^-1, или я опять не про то говорю?
Аноним 02/02/16 Втр 22:36:53 #70 №335411 
>>335310
Платонист закукарекал.
Аноним 02/02/16 Втр 22:37:01 #71 №335412 
>>335409
Ты не обладаешь необходимым уровнем знаний для исчерпывающего и внятного ответа. Ты как астролог, даже хиромант. Твоим следующим действием будет призыв сжигать еретиков
Аноним 02/02/16 Втр 22:44:39 #72 №335414 
>>335357
https://www.coursera.org/learn/calculus1
Аноним 02/02/16 Втр 22:47:02 #73 №335415 
>>335395
Ты либо неприлично толстый, либо непроходимо тупой.
Аноним 02/02/16 Втр 22:47:46 #74 №335416 
>>335412
Съеби уже в /ph какой-нибудь.
Аноним 02/02/16 Втр 23:07:02 #75 №335419 
>>335416
Анон всю твою натуру через интернет вычислел. Прогнать, изгнать, колесовать, сжечь на костре, разстрелять как бешеных собак
Аноним 02/02/16 Втр 23:09:52 #76 №335420 
>>335410
H^2 = H, так как H группа и, следовательно, содержит единицу.
Не про то. aH=Ha <-> aHa^-1=H <-> aha^-1 \in H для любого h.
Аноним 02/02/16 Втр 23:28:10 #77 №335422 
>>335412
>>335419
Какая жалкая попытка семёнить. Просто всем неинтересно общаться с дауном, а тебе будет неинтересно общаться с умными людьми, так как ты - даун. Вот тебе и предложили съебать. Можешь оставаться, конечно, благородные доны будут время от времени снисходить до того, чтобы поссать тебе в рот, например.
Аноним 02/02/16 Втр 23:29:30 #78 №335423 
>>335420
>aH=Ha <-> aHa^-1=H <-> aha^-1 \in H
Почему тогда просто не написать aH=Ha принадлежит H? Почему в определении именно сопряженные элементы?
knuebok 02/02/16 Втр 23:38:42 #79 №335425 
>>335423
aH - это множество, состоящее из всех элементов H умноженных слева на а, соответственно, Ha - это множество, состоящее из всех элементов H, умноженных справа на a, равенство: aH = Ha это равенство множеств, которое, если раскручивать, значит буквально следующее: для любого a из G, существуют такие h1 и h2 из H, что a h1 = h2 a. отсюда a h1 a^-1 = h2, что значит просто то, что a h a^-1 принадлежит H для любых a из G и h из H.
knuebok 02/02/16 Втр 23:40:33 #80 №335426 
>для любого a из G, и h1 из H существует такое h2 из H, что a h1 = h2 a.
fix
Аноним 03/02/16 Срд 00:01:07 #81 №335428 
>>335425
Лол, спасибо, у меня как-то по-другому все в сознании работает. Я бы иной формализм создал, будь я на месте основателей теории групп. По-моему, можно все проще сделать. Но суть теперь понятна. Еще раз благодарю, а то мучаюсь с этими вопросами уже добрую неделю.
Аноним 03/02/16 Срд 00:01:09 #82 №335429 
ГООЛ ВАРДИ! ХА-ХА-ХА-ХА!
Аноним 03/02/16 Срд 00:01:35 #83 №335430 
>>335429
Ой, блядь, я еблан.
Аноним 03/02/16 Срд 00:02:03 #84 №335431 
>>335430
Обоснуй с выкладками.
Аноним 03/02/16 Срд 00:46:20 #85 №335438 
>>335428
На самом деле гораздо лучше посмотреть на факторы всяких групп по всяким нормальным подгруппам, потом посмотреть на ненормальные подгруппы и подумать, почему не пройдёт номер точно так же их отфакторизовать.
Аноним 03/02/16 Срд 02:04:11 #86 №335442 
>>335184 (OP)
Почему ноль и нуль считаются тождественными понятиями. Ведь ноль, если прочитать наоборот будет лон->лоно, разве лоно пустое? Нет оно полное. А если нуль прочитать наоборот будет лун->луна->лунка, вот это ничто, может быть по этому луну назвали луной так как она внутри полая? Почему математики не видят в этих словах разницы?
Аноним 03/02/16 Срд 02:49:54 #87 №335450 
>>335184 (OP)
Как доказать, что в хаусдорфовом пространстве одноточечное множество замкнуто?
Аноним 03/02/16 Срд 03:52:06 #88 №335456 
>>335339
Спасибо за содержательный ответ.
Аноним 03/02/16 Срд 04:32:59 #89 №335458 
>>335402
Бесконечность не число, это концепция, идея - возникшая из-за неспособности нашего мозга полностью понять реальность, даже имея компьютеры. Человек, когда изучает мир, часто пытается делить объекты на более мелкие составные части, но это приводит часто к абсурду. Также и с отрезками из твоего вопроса, состоящими из составных частей, самой маленькой и неделимой частью считается точка. На первый взгляд это невозможно, но на самом деле АВ имеет бесконечно точек как и CD, но бесконечность бесконечности рознь, бывает так, что одна бесконечность больше другой бесконечности. Пусть длина АВ равна 1см, а CD равна 2см. Загадай отрезок длиной от 0 до 1см - вероятность того, что ты загадаешь отрезок помещающийся в АВ, такая же, как и вероятность загадывания отрезка помещающегося в CD. Загадай отрезок длиной от 0 до 2см - вероятность загадывания отрезка помещающегося в CD будет большей, а значит CD включает в себя большее количество составных частей и бесконечность точек CD > АВ. В начале 20-го века ввели понятие МЕРА ОТРЕЗКА, МЕРА МНОЖЕСТВА которые какбы интуитивно связаны с их размерами. В общем ищи в google…
knuebok 03/02/16 Срд 04:39:47 #90 №335459 
>>335450
Возьмём точку p и точку x. Возьмём окрестность Ux, которая не содержит точки p. Объединение всех Ux x \in \Omega, x!=p даст как раз дополнение до {p}.
Аноним 03/02/16 Срд 04:48:07 #91 №335460 
>>335459
Хм, и правда. Я тупой.
Аноним 03/02/16 Срд 05:07:52 #92 №335462 
>>335184 (OP)
>Некоторые люди ведут войну с аксиомой выбора, утверждая, что её следствия парадоксальны
Какие, например?
Аноним 03/02/16 Срд 05:38:17 #93 №335464 
>>335462
Вот, например, прикольное возражение.
http://lj.rossia.org/users/muda/5782.html
Аноним 03/02/16 Срд 13:13:21 #94 №335536 
>>335402
>>335407
Обоже, какой даун
Аноним 03/02/16 Срд 13:15:15 #95 №335537 
>>335458
> Бесконечность не число, это концепция, идея - возникшая из-за неспособности нашего мозга полностью понять реальность, даже имея компьютеры.
Блять, /ph/ протек.
Ууууу блять, как же вы мне противны.
Аноним 03/02/16 Срд 13:37:29 #96 №335540 
>>335184 (OP)
5^Пи = 1√5^Пи
Аноним 03/02/16 Срд 13:49:03 #97 №335546 
>>335540
У матанодаунов число пи не определено. Это физекам 3.1415926 уже с головой хватает чтобы длинну окружности атома ксенона рассчитывать, а маняматикам подавай АБСАЛЮТНАЮ ТОЩНАСТ
Аноним 03/02/16 Срд 13:52:01 #98 №335550 
>>335184 (OP)
5^Pi = 156.50 - 157.10
Аноним 03/02/16 Срд 13:57:53 #99 №335556 
>>335442
Отвечайте на мой вопрос, бляди.
Аноним 03/02/16 Срд 14:38:45 #100 №335577 
>>335556
>Отвечайте
Ответил.
Аноним 03/02/16 Срд 14:48:17 #101 №335581 
>>335556
Ответил тебе за щеку, проверяй.
sageАноним 03/02/16 Срд 14:48:32 #102 №335582 
>>335546
>У матанодаунов число пи не определено.
Нда.
Аноним 03/02/16 Срд 14:48:32 #103 №335583 
>>335577
Где?
Аноним 03/02/16 Срд 14:54:53 #104 №335584 
14545004935330.png
А поясните за ZFC.
1) Вся математика сводится к исчислению предикатов на основе аксиом ZFC, или что-то не сводится?
2) Возможны ли другие подобные системы аксиом, или ZFC уникальна?
Аноним 03/02/16 Срд 15:02:40 #105 №335587 
>>335407
Мощность отрезка-количество точек принадлежащих этому отрезку, и она имеет мощность континуума.
Длинна отрезка-функция от координат концов отрезка.
Аноним 03/02/16 Срд 15:02:48 #106 №335588 
>>335537
Противен здесь только ты, быдло.
Аноним 03/02/16 Срд 15:19:36 #107 №335592 
>>335584
Если ты хочешь использовать теорию категорий, то ZFC не подойдет.
Можешь почитать про NBG
https://en.wikipedia.org/wiki/Von_Neumann%E2%80%93Bernays%E2%80%93G%C3%B6del_set_theory
Аноним 03/02/16 Срд 15:28:27 #108 №335593 
14545025079510.jpg
>>335592
А NBG актуальна во всех вообще случаях, или есть что-то что не сводится к ней?
Аноним 03/02/16 Срд 15:30:56 #109 №335594 
>>335583
Там.
Аноним 03/02/16 Срд 15:31:21 #110 №335595 
>>335593
Не знаю, я не разбираюсь в основаниях.
Аноним 03/02/16 Срд 15:54:56 #111 №335601 
>>335593
Во всех.
Аноним 03/02/16 Срд 15:57:10 #112 №335602 
14545042303530.jpg
Аноним 03/02/16 Срд 16:01:18 #113 №335603 
>>335582
Ну давай пиши мне ТОЧНОЕ значения пи.
Аноним 03/02/16 Срд 16:02:12 #114 №335604 
>>335603
3
Аноним 03/02/16 Срд 16:02:41 #115 №335605 
>>335602
В чем математический эффект?
Аноним 03/02/16 Срд 16:03:17 #116 №335606 
>>335604
Высрался тебе в глотку. И НЕ СМЕЙ ВЫТИРАТЬСЯ, МРАЗЬ!!!!!!!
Аноним 03/02/16 Срд 16:17:23 #117 №335607 
>>335604
как грубо!
Аноним 03/02/16 Срд 16:23:15 #118 №335608 
>>335605
Меньшее количество постов надо будет скрывать, экономия времени
Аноним 03/02/16 Срд 16:28:38 #119 №335609 
>>335603
Выбери любой ряд, сумма которого равна пи.
Аноним 03/02/16 Срд 16:31:03 #120 №335610 
>>335609
Еще раз в глотку испражнился.
Аноним 03/02/16 Срд 16:49:14 #121 №335612 
>>335608
Не понял.
Аноним 03/02/16 Срд 18:19:00 #122 №335624 
14545127410720.png
Аноны, поясните за импликацию как так получается так в последнем примере 0 0 по идее там три нуля должно быть?
Аноним 03/02/16 Срд 18:24:12 #123 №335625 
>>335624
По какой ещё идее? Ты с определением её разберись.

НЕ(0)+0=1+0=1
Аноним 03/02/16 Срд 18:34:42 #124 №335626 
>>335625
ну тогда второй пример
почему так? Правдивое основание и ложный вывод, дает правду
1 0 A=>B дает правду?
Выглянуло солнце => Стало темно и это правда
Аноним 03/02/16 Срд 19:03:00 #125 №335629 
>>335624
У тебя таблица истинности от обратной импликации вроде. Ноль должен быть во второй строке (и только).
Аноним 03/02/16 Срд 19:39:18 #126 №335630 
>>335407
Почитай что нибудь по теории функции и меры, потом возвращайся.
Аноним 03/02/16 Срд 19:44:14 #127 №335631 
>>335419
>раЗстрелять
Портфель собрал?
sageАноним 03/02/16 Срд 20:06:27 #128 №335636 
>>335610
Почему?
Аноним 03/02/16 Срд 20:09:25 #129 №335638 
>>335636
Потому что ты даун
Аноним 03/02/16 Срд 20:26:31 #130 №335643 
>>335603
Ну давай сформулируй мне, что такое точное значение.
Аноним 03/02/16 Срд 20:28:40 #131 №335644 
>>335643
Не буду. Иди нахуй, ты просто хочешь говном покидаться.
Аноним 03/02/16 Срд 20:38:22 #132 №335645 
>>335592
На самом деле это не так принципиально - есть по крайней мере три разных подхода к формализации теории категорий в теории множеств (и один из них проходит собственно в ZFC):
1) Подход, когда большие категории понимаются, как классы, он обычно формализуется в теории множеств Маклейна (чуть усиленная NBG).
2)Подход на основе вселенных, когда аксиоматически полагается, что имеется семейство вложенных вселенных множеств и вместо рассмотрения, например, категории всех групп рассматривается категория всех групп в некоторой вселенной и эта категория оказывается множеством с точки зрения вышестоящей вселенной (формализуется в ZFC + аксиома вселенных Гротендика).
3)Замена вселенных Гротендика из 2) на немного более слабые, существование которых доказуемо в чистой ZFC (принцип отражений).
Аноним 03/02/16 Срд 21:00:29 #133 №335648 
Всем привет. Почему сущности равны сами себе? Например, почему 2=2?
sageАноним 03/02/16 Срд 21:01:54 #134 №335649 
>>335648
----> /ph/
Аноним 03/02/16 Срд 21:08:26 #135 №335651 
>>335649
А мне интересна позиция математиков. Наверняка этому есть обоснование. Может просто аксиоматическое, а может откуда-нибудь следующее.
Аноним 03/02/16 Срд 21:10:32 #136 №335652 
>>335651
Скинул доказательство тебе за щечку, следую за ним.
Аноним 03/02/16 Срд 21:12:30 #137 №335653 
Скиньте книжку по математике с первого до 11 класса, чтобы там было все популярно описано и можно было сыночке-корзиночке все показать и рассказать.
Аноним 03/02/16 Срд 21:12:59 #138 №335654 
>>335651
http://phenomen.ru/public/dictionary.php?article=1119
Аноним 03/02/16 Срд 21:18:03 #139 №335655 
>>335652
У-тю-тю, какой ты мамкин злобный омежка. Даже злиться на тебя не хочется.
Аноним 03/02/16 Срд 21:20:17 #140 №335656 
>>335654
Интересно, но как-то неубедительно. Все это нужно еще доказать.
Аноним 03/02/16 Срд 21:20:47 #141 №335657 
>>335653
Трёхтомник "Основания математики" Рассела.
Аноним 03/02/16 Срд 21:22:14 #142 №335658 
>>335656
>как-то неубедительно.
Что именно?
Аноним 03/02/16 Срд 21:25:59 #143 №335660 
>>335656
Это на самом деле стандартная позиция, которая, в частности, отражена в аксиоме объемности в теории множеств. Учитывая то, что математика, по существу, культурный феномен, если что-то воспринимается математическим сообществом, как аксиома, то это она и есть. Аксиомы не подлежат доказательству.
Аноним 03/02/16 Срд 21:27:59 #144 №335661 
>>335657
Семилетняя личинка поймет то что там написано?
Да и три тома это как то слишком для ребенка, как по мне.
Аноним 03/02/16 Срд 21:30:44 #145 №335662 
>>335661
Да, конечно, ведь там все действительно подробно расписано, единственная проблема что математика там явно не доходит до 11 класса (в лучшем случае он относится к 3-4).
Аноним 03/02/16 Срд 21:31:13 #146 №335663 
Ребята, какое наиболее точно название объекта, поддерживающего операции сложения и умножения (не обязательно так называющиеся, но имеющие такие же свойства).
Аноним 03/02/16 Срд 21:31:42 #147 №335664 
>>335663
пример этих объектов: числа, многочлены
Аноним 03/02/16 Срд 21:33:55 #148 №335665 
>>335644
Это ты хочешь покидаться. Тебе предложили точное значение пи, ты сказал, что это неверно. Вполне закономерный вопрос я задал.
Аноним 03/02/16 Срд 21:35:02 #149 №335666 
>>335648
Потому что это один и тот же символ.
Аноним 03/02/16 Срд 21:35:37 #150 №335667 
>>335662
Три тома расписывать уровень третьего класса? Там что, абстракции уровня бурбаки?
Аноним 03/02/16 Срд 21:39:46 #151 №335668 
>>335667
Трактат Бурбаки, по существу, идеологическое продолжение книги Уайтхеда Рассела. При этом, надо отметить, что он написан, с учетом ошибок предшественников, принципиально более человечным языком и опусканием куда большего числа формальных деталей.
Аноним 03/02/16 Срд 21:40:36 #152 №335669 
>>335658
То, что если между двумя объектами нет различий, то они тождественны.
Аноним 03/02/16 Срд 21:40:40 #153 №335670 
>>335668
Ясно.
А для детей-школяров то что то есть?
Аноним 03/02/16 Срд 21:50:47 #154 №335673 
>>335669
Это определение тождественности.
Аноним 03/02/16 Срд 21:52:55 #155 №335674 
>>335648
По определению равенства. Мы пишем a = b, если симолы a и b обозначают один и тот же объект.
Аноним 03/02/16 Срд 21:54:21 #156 №335675 
>>335674
Не равенства, а тождества.
Аноним 03/02/16 Срд 22:12:32 #157 №335677 
>>335670
Трёхтомник "Основания математики" Рассела.
Аноним 03/02/16 Срд 22:14:03 #158 №335679 
>>335675
Чем равенство отличается от тождества?
Аноним 03/02/16 Срд 22:15:37 #159 №335680 
>>335673
Определение не является доказательством. Значит все-таки просто взяли и и сказали, что оно будет так. Интересно как изменится математика, если отбросить эту догму.
Аноним 03/02/16 Срд 22:16:00 #160 №335681 
>>335679
Тождество - равенство истинное при всех значениях переменных.
Аноним 03/02/16 Срд 22:16:44 #161 №335682 
>>335677
Ты сам сказал что там до 3 класса только поясняют все. А до 11 класса что делать? Сразу дискриминантам его учить с матрицами?
Аноним 03/02/16 Срд 22:16:46 #162 №335683 
>>335680
Это не догма.
Аноним 03/02/16 Срд 22:18:41 #163 №335685 
>>335683
Все, что объявляется истиной без доказательств можно назвать догмой.
Аноним 03/02/16 Срд 22:19:31 #164 №335687 
>>335685
Нет. Разберись с определениями понятий, которые ты употребляешь.
Аноним 03/02/16 Срд 22:20:00 #165 №335688 
>>335687
Обычные ноотроп, что такого то? Все математики такие употребляют.
Аноним 03/02/16 Срд 22:20:45 #166 №335689 
>>335682
Трактат Бурбаки, по существу, идеологическое продолжение книги Уайтхеда Рассела. При этом, надо отметить, что он написан, с учетом ошибок предшественников, принципиально более человечным языком и опусканием куда большего числа формальных деталей.
Аноним 03/02/16 Срд 22:21:02 #167 №335690 
>>335685
Твои постановки вопросов по своему духу явно относятся не к этому треду, а к соседнему >>295488 (OP)
Аноним 03/02/16 Срд 22:21:28 #168 №335691 
>>335687
Лол, употребляю те понятия, которые мне удобны. С чего это мне отчитываться перед кем-то?
Аноним 03/02/16 Срд 22:24:51 #169 №335692 
>>335689
Ясно. Скрыл.
knuebok 03/02/16 Срд 22:34:04 #170 №335693 
>>335601
Особенно в тех, когда мы хотим рассмотреть категорию всех классов с морфизмами - отображениями.
>>335663
Кольцо.
Аноним 03/02/16 Срд 22:52:00 #171 №335696 
>>335692
Если серьёзно, то, надеюсь, Вы знаете, что школьная математика - не математика вовсе, а отработка простейших навыков и умений работать с тривиальными методами. Вы желаете отработать с ребёнком именно их? Тогда необходима простейшая теория и как можно больше практики. Для этих целей подходит сборник "Пособие для поступающих в вузы" под авторством Модёнова. Все арифметические законы Вам придётся объяснять самому, а само введение в книге требует знаний на уровне пятого класса - тоже ваша работа.
Но на самом деле важно также подготовить не только теоретическую базу, но и умение рассуждать логически. Для этих целей подходит серия из двух книг "Планиметрия. Пособие для углубленного изучения математики" и "Стереометрия. Пособие..." - не предполагает изначальных алгебраических знаний и, по-моему, намного лучше предлагаемого сейчас в школах курса под авторством Атанасяна.

Но если ребёнку изначально не очень интересен абстрактный мир математики, ничего не получится.
Аноним 03/02/16 Срд 23:23:09 #172 №335699 
>>335693
А что не так с категорией всех классов?
knuebok 03/02/16 Срд 23:24:43 #173 №335700 
>>335699
Её носитель не может быть даже собственным классом.
Аноним 03/02/16 Срд 23:26:50 #174 №335701 
>>335648
Это и аксиома/постулат и прямой результат минимально здоровой логики. Доказательством верности этой аксиомы служат очевидные факты нашей каждодневной жизни т.е. верность этой аксиомы доказать очень легко и никто даже не требует доказательств. Например: говоришь ребенку, что не хорошо прятать в кармане 2 слитка золота. Ребенок тут же говорит что у него нет вообще никакого золота, т.е. 2 не равно 0, а 0=0, а был бы у ребенка 1 слиток золота, то ребенок сам бы тебе сказал 1 не равен 2
knuebok 03/02/16 Срд 23:35:47 #175 №335702 
14545317479530.gif
>>335648
Аноним 04/02/16 Чтв 01:23:23 #176 №335710 
>>335648
В архивах соросовского оразовательного журнала есть статья "Тождество". Лежит в интернетах. Гугли.
Аноним 04/02/16 Чтв 01:37:49 #177 №335711 
Я спать.
sageАноним 04/02/16 Чтв 01:58:25 #178 №335712 
>>335701
Болтовня.
Аноним 04/02/16 Чтв 02:17:35 #179 №335713 
>>335712

Честно говоря я с тобой согласен и в этой жизни все болтовня и вся математика и физика и химия... Чем больше изучаешь, тем больше понимаешь, что ничего на самом деле не знаешь. Вроде столько ученых, но никто не может четко предсказать среднюю цену на нефть/золото... в 2016; не могут даже точно сказать куда и когда грохнется затапливаемая космическая станция.
Аноним 04/02/16 Чтв 10:12:24 #180 №335747 
>>335696
Я это понимаю, но я лично могу вспомнить из школьного курса только дискриминант, а это уже уровень 8-9 класса вроде. Что там до него проходится я не помню.
Посмотрю что там за книжечки.
И мне надо просто научить ребенка самой базовой математике, чтобы потом не пришлось объяснять почему квадратный многочлен связан с параболой и что скрывается за понятием корня.
Аноним 04/02/16 Чтв 10:21:38 #181 №335749 
>>335747
Базовая математика - это как базовая физика. Всё настолько упрощено, что ребёнок потом, когда узнает правду, просто запутается.
Аноним 04/02/16 Чтв 10:29:45 #182 №335750 
>>335747
Зачем учить ребёнка этой фигне? В школе ему и без тебя голову испортят. Учи его теории множеств и другим полезным вещами.
Аноним 04/02/16 Чтв 10:34:50 #183 №335752 
>>335747
Если ты сам знаешь школьную математику на хорошем уровне, то сможешь её рассказать сам, без всяких книжечек. (максимум что, можно смотреть на идеи для интересных задач), а если не знаешь, то и книжечка не поможет - выйдет полное уебанство, лучше отдай его в какой-нибудь хороший олимпиадный мат. кружок.
Аноним 04/02/16 Чтв 10:39:57 #184 №335753 
>>335747
Стандартные шени с гельфандами:
Гельфанд Шень Алгебра
Шень Геометрия
Аноним 04/02/16 Чтв 11:52:30 #185 №335760 
>>335546
Пи = Пи/1, а дальше,
по первой формуле с пикчи ОП-поста.

Для выражения, не обязательна ОПСАЛЮТНОЙА ТОШНОСТЬ.
У меня вопрос Аноним 04/02/16 Чтв 12:01:12 #186 №335762 
14545764726910.jpg
В известной программе сказано:
>Геометрия на верхней полуплоскости (Лобачевского). Свойства инверсии. Действие дробно-линейных преобразований.
Уважаемые господа, что конкретно под этим понимается? Какие главы в какой книжке читать? Поясните по-хардкору.
Аноним 04/02/16 Чтв 12:45:44 #187 №335767 
>>335750
Двачую
Аноним 04/02/16 Чтв 13:01:51 #188 №335770 
>>335762
Есть брошюра Прасолова: https://vk.com/doc41446009_257315839?hash=4fe79ece33d870910e&dl=c91ee440edf1890ea7 но вообще я бы не советовал задрачиваться - юзелесс хуйня с передозом тригонометрии.
Аноним 04/02/16 Чтв 14:45:01 #189 №335791 
14545863018290.jpg
Вот смотрите.
1) Вся математика сводится к логике первого порядка на основе аксиом, скажем, NBG.
2) Все правила вывода в исчислении предикатов - чисто механическая задача.
3) Доказательство любой теоремы - чисто механическая процедура в пруф ассистантах / ЯП с зависимыми типами.
Собственно, идея:
1) Берем клеточные автоматы, много клеточных автоматов, прописываем в них правила исчисления предикатов на основе заданной аксиоматики.
2) Нагенерированные сетями клеточных автоматов теоремы верифицируем в пруф ассистантах.
3) То, что приводит к противоречиям и т.п. удаляем, оставляем доказанные непротиворечивые.
4) Доказанные теоремы пробиваем по всем поисковым системам краулерами, те что уже существуют и доказаны, удаляем.
5) Остаются теоремы и прочие построения, которые во-первых, верны, во-вторых, не встречаются в современной математике.
6) ...
7) Выгода?
Аноним 04/02/16 Чтв 14:49:09 #190 №335794 
>>335791
>7) Выгода?
Нулевая.
Аноним 04/02/16 Чтв 15:05:37 #191 №335798 
>>335791
Я кстати давно о таком думал. Подвох небось в нехватке памяти для хранения такого объема комбинаторики.
Аноним 04/02/16 Чтв 15:09:29 #192 №335800 
>>335798
Подвох в том, что главная ценность теорем - их интуитивный смысл. Всё то облако понятий, знаний, эмоций, которое связано с теоремой. Компьютер пока что не умеет писать осмысленные тексты, так что генерировать математику компьютером не получится. Компьютер может генерировать только строчки символов по заданным правилам. Но что с этого толку? Любая строчка символов является теоремой в некоторой подходящей системе аксиом. Но ведь не все строки символов эквивалентны.
Аноним 04/02/16 Чтв 15:23:32 #193 №335803 
>>335800
>строчки символов по заданным правилам
В этом суть любого математического построения.
>главная ценность теорем - их интуитивный смысл.
Смысл метематического построения не зависит от того как оно получено. Более того, правильное математическое построение в смысле вообще не нуждается. Главное - непротиворечивость, еще бурбаки об этом писали.
>Всё то облако понятий, знаний, эмоций, которое связано с теоремой.
У тебя просто магическое мышление.
Аноним 04/02/16 Чтв 15:31:53 #194 №335806 
>>335800
Я уже вижу как все математики зассали перед мощью компьютерных числодробилок. Данные удобнейшие для компрессии, огромные процессинговые фермы, йобабайты данных, фильтры, критерии, комбинаторика и оптимизация, оптимизация, оптимизация, оптимизация... И самую могущественную математику творят Васяны, прочитавшие книжку "C++ за 21 день".
Аноним 04/02/16 Чтв 15:36:28 #195 №335807 
>>335806
А так и будет. Жалкие людишки математику давно не тянут. Сколько там веков была открытой проблема теоремы Ферма? Потом, АВС-гипотеза. С этим еще смешнее, один японец доказал, а кроме него никто в доказательство вкурить не может уже четвертый год. Нахуй так жить вообще? А ведь АВС-гипотеза и теорема Ферма - жалкая хуйня про натуральные числа, которые школьники в 1 классе проходят. По-настоящему сложная математика еще даже не начиналась.
Аноним 04/02/16 Чтв 15:41:26 #196 №335808 
>>335791
Загвоздка не в "интуитивном смысле теорем" (который окончательно проёбан ещё в XIX веке кмк).

И не в нехватке памяти для объёма комбинаторики.

А в том, что теорем будет овердохуя и они будут бесполезными до шизоидности. Условно говоря, алгоритм тебе выдаст миллиард функций вида f(x^+y^2)=0 и докажет — рассматривая каждую по отдельности — что они все суть окружности, хотя при умении абстрагироваться и так ясен хуй, что это всё окружности.

И уже на сотой f(x^2+y^2)=0 ты плюнешь на это дело и пойдёшь двачевать.

Я этой темой не занимался (только что-то слышал краем уха), так что это просто домыслы диванного эксперта. Было бы приятно оказаться неправым.
Аноним 04/02/16 Чтв 15:44:37 #197 №335809 
>>335808
Ты будто из докомпьютерного века. Почему алгоритм не может занести эти функции в класс? Не создать правило? Не объединить правила?
Аноним 04/02/16 Чтв 15:45:52 #198 №335811 
>>335807
Можно выдумывать сколь угодно говнотеорем, которые будет понимать только автор. В википедии это называется ОРИСС. Математика ущербна. А ты ещё ущербнее чем математика.
Аноним 04/02/16 Чтв 15:46:51 #199 №335812 
>>335811
>Математика ущербна.
Но ведь ущерб здесь только ты. Пиздуй в /ph или откуда ты вылез.
Аноним 04/02/16 Чтв 15:47:39 #200 №335814 
>>335803
>еще бурбаки об этом писали
Сразу видно человека, который не читал бурбаков.
>в смысле вообще не нуждается
В таком случае вот тебе формальная система.
1. Символ ы есть строка.
2. Конкатенация двух строк есть строка.
3. Доказательный текст есть набор строк, записанных друг за другом, так, что каждая очередная строка есть либо аксиома, либо конкатенация каких-либо двух предыдущих строк.
4. Теоремы есть строки, встречающиеся в доказательных текстах. Доказательный текст, в котором встречается теорема, называется доказательством теоремы.
5. Строка ы - аксиома.

Ты можешь, в полном соответствии со своей философией, посвятить жизнь построению теорем в этой формальной системе - ведь для тебя нет разницы, какой формальной системой пользоваться. Я могу первые несколько теорем вывести за тебя.

Теорема 1. ы. Доказательство: ы.
Теорема 2. ыы. Доказательство: ы ы ыы.
Теорема 3. ыыы. Доказательство: ы ы ыы ыыы.
...
И так далее. Думаю, в день ты сможешь открывать тысячи теорем. Вперёд, удачи.
Аноним 04/02/16 Чтв 15:50:13 #201 №335817 
>>335814
>Ты можешь, в полном соответствии со своей философией, п
Но мне не нужна твоя фейлософия. И я ясно написал, какую систему аксиом и какие правила вывода из них имею в виду.
>>335814
>Сразу видно человека, который не читал бурбаков.
Именно.
Аноним 04/02/16 Чтв 15:50:23 #202 №335818 
>>335812
Пиздуй в /em, там на языки дрочат.
Аноним 04/02/16 Чтв 15:52:10 #203 №335820 
>>335814
Теперь докажи эквивалентность твоей фс и NBG -- и дело в шляпе.
Аноним 04/02/16 Чтв 15:54:16 #204 №335822 
>>335820
А зачем? Ведь смысл никого не интересует. А раз смысл не важен, то какая разница, какую формальную систему изучать.
Теорема 4. ыыыы. Доказательство: ы ы ыы ыы ыыыы.
Аноним 04/02/16 Чтв 15:55:43 #205 №335823 
>>335809
> Не создать правило?
Потому что тебе придётся вручную набить весь класс бесполезных шизодных теорем, чтобы программа их отсеивала.

А что-то мне подсказывает, что их таких гораздо больше, чем хороших и нужных. Сначала ты скажешь фильтру: окей, не надо доказывать про круги. Потом: не надо про круги в 3-х измерениях. Четырёх. Потом будут не круги, а просто какая-нибудь ебала. А потом ещё. И так до бесконечности.
Аноним 04/02/16 Чтв 15:55:50 #206 №335824 
>>335814
Более того, та хуйня что ты описал, Л-системы, или системы Линденмайера, давно используют, правда, поумнее чем это сделал ты.
>>335822
>то какая разница, какую формальную систему изучать.
Я даже написал, какая разница. Исходный пост >>335791 ты либо не читал, либо, что скорее всего, не понял.
Аноним 04/02/16 Чтв 15:57:11 #207 №335825 
>>335823
Давай скажем проще - ты просто понятия не имеешь о клеточных автоматах.
Аноним 04/02/16 Чтв 15:59:11 #208 №335826 
>>335824
Кое-кто выше написал:
>правильное математическое построение в смысле вообще не нуждается. Главное - непротиворечивость
Объясни с такой точки зрения, чем моя формальная система (я нареку её ЫЫЫ) хуже, чем NBG.
Аноним 04/02/16 Чтв 15:59:17 #209 №335827 
>>335822
Смысл это жалкое творение эволюции, как и сам человек. Тебя окружает лишь белый шум. Твои глаза, уши, мозг -- инструмент, выделяющий из шума определенной структуры этот самый смысл. Если бы наши предки кушали радиоволны, то мы бы видели радиоволны, смекаешь? Так почему мы не можем создать такой же инструмент для математики? Пусть он будет примитивный, как глаз моллюска, но ведь он тоже сможет эволюционировать, если подвергнуть его отбору. Для человека это более рациональный путь, ведь создавая самого себя он руководствуется по сути такими же этими случайно возникшими принципами (аминокислотами, клетками) как инструкцией.
Аноним 04/02/16 Чтв 16:00:48 #210 №335828 
>>335806
Лол, эти сказки еще Гильберт в начале прошлого века рассказывал и тогда это было куда убедительнее, чем сейчас.
Аноним 04/02/16 Чтв 16:00:51 #211 №335829 
>>335825
>ты просто понятия не имеешь о клеточных автоматах.
Клеточный автомат это либо Машина Тьюринга, только в профиль, либо сильно урезанная машина Тьюринга.

Да, кстати, объясни, зачем ты к своему выводителю теорем прикрутил это стародавнюю игрушку студентов-первокурсников? Как они решат тебе проблему, описанную в >>335823
?
Аноним 04/02/16 Чтв 16:01:36 #212 №335830 
>>335823
Когда ты говоришь "класс" ты же что-то подразумеваешь под этим понятием? Наверное, каким-то правилом руководствуешься, чтобы выделить класс. Я тебе расскажу про сепульки и ты выделишь из них класс, каким-то чудным образом.
Аноним 04/02/16 Чтв 16:01:47 #213 №335831 
>>335826
>Объясни с такой точки зрения, чем моя формальная система (я нареку её ЫЫЫ) хуже, чем NBG.
Тем, что практической ценности не представляет.
sageАноним 04/02/16 Чтв 16:02:03 #214 №335832 
>>335827
Всё, пиздуй в /ph, изобретатель.
Аноним 04/02/16 Чтв 16:02:18 #215 №335833 
>>335826
https://en.m.wikipedia.org/wiki/Soundness
https://en.m.wikipedia.org/wiki/Completeness_(logic)
Аноним 04/02/16 Чтв 16:02:47 #216 №335834 
>>335831
Практическая ценность - это, внезапно, смысл. Выходит, осмысленность всё же нужно учитывать?
Аноним 04/02/16 Чтв 16:03:01 #217 №335835 
>>335831
Что значит «практической ценности»?
Аноним 04/02/16 Чтв 16:03:25 #218 №335837 
>>335834>>335835
-> /ph
Аноним 04/02/16 Чтв 16:03:30 #219 №335838 
>>335834
>Практическая ценность - это, внезапно, смысл.
Нет.
Аноним 04/02/16 Чтв 16:04:41 #220 №335839 
>>335830
>Когда ты говоришь "класс" ты же что-то подразумеваешь под этим понятием?
Да, я подразумеваю бесконечный список лютой хуиты, которую непонятно как отсеивать.

И ты даже не попытался решить проблему просеивания теорем.
Аноним 04/02/16 Чтв 16:06:18 #221 №335840 
>>335832
Это не философия, это просто констатация факта. Фотоны с монитора попадают тебе на рецептор, а дальше путем сложных манипуляций ты сначала неосознанно выделяешь формы, цвета, объем, после объединяешь это в более сложные структуры, которые для тебя речь, выискиваешь в этом некую "философскую подоплеку". Но это же просто набор фотонов, разве способна сраная нейросеть на что-то подобное? Ей ведь не хватит мощности и памяти на такое!
Аноним 04/02/16 Чтв 16:07:28 #222 №335842 
>>335838
Не нет, а да. Смысл теории - это, по определению Клини, всё то, что не описано в метатеории формальной теории явно. Практическая ценность не описана ни в метатеории NBG, ни в метатеории ЫЫЫ, поэтому практическая ценность - смысл.
В ЭТОМ ВЕСЬ МАТАНЧ-ТРЕНД: Аноним 04/02/16 Чтв 16:07:38 #223 №335843 
14545912583820.jpg
...три годных сообщения, написанных много думавшими людьми в течение недели, и взрыв говна, настроченного шизиком за 15 минут, которое похоронило всё.
Аноним 04/02/16 Чтв 16:09:12 #224 №335845 
>>335842
Но смысл – не практическая ценность.
Аноним 04/02/16 Чтв 16:10:31 #225 №335846 
>>335845
Практическая ценность - смысл.
Допустим, смысл не имеет значения. Тогда и практическая ценность, как часть смысла, не имеет значения.
Но практическая ценность имеет значение. Значит, допущение неверно.

Засим обтекай.
Аноним 04/02/16 Чтв 16:10:45 #226 №335847 
>>335839
Ты же обнаружил в круге, измерении, прямой, функции, etc, какую-то похожесть. Некоторые похожи алгебраической записью, некоторые графиком, некоторые производной, да дохуя критериев. Математическая интуиция это лишь набор фактов в голове. Вспомнил что-то и связал. А у компьютерного фильтра в качестве источника фактологии весь arXiv.org.
Аноним 04/02/16 Чтв 16:12:07 #227 №335848 
>>335847
>какую-то похожесть
Формализация этой "похожести" эквивалентна созданию сильного ИИ.
Аноним 04/02/16 Чтв 16:12:47 #228 №335849 
>>335847
Свечи перед пультом.
Аноним 04/02/16 Чтв 16:13:11 #229 №335850 
>>335846
>>335845
sageАноним 04/02/16 Чтв 16:15:44 #230 №335851 
>>335847
>Ты же обнаружил
>imply Значит, и компуктер обнаружит

> интуиция это лишь набор фактов в голове.

Ты себя со стороны совсем не видишь, поехавший?
sageАноним 04/02/16 Чтв 16:17:20 #231 №335852 
>>335847
>да дохуя критериев.
Удачи тебе их все формализовать, увязать и замкнуть на себя для дальнейшего автоматического расширения и развития.
Аноним 04/02/16 Чтв 16:18:18 #232 №335853 
>>335848
В общем смысле -- да. Но я не уверен в том, что все существующие в публикациях математические объекты абсолютно изучены и не могут принести пользу.
Аноним 04/02/16 Чтв 16:23:36 #233 №335854 
Вы дауны? Нахуя вы зеленего дурака кормите? Зарепортили и концы в воду.
Аноним 04/02/16 Чтв 16:23:48 #234 №335855 
>>335851
Разговор прежде всего вероятностях, а не о причинно-следственных связях. Какова вероятность того, что сраная клетка с рецептором эволюционируют за пару миллиардов лет до человека, способного в этого разговор? У нас же есть весьма неплохие возможности ускорить этот процесс и начать с более развитого организма, чем клеточный автомат, аналогичный клетке.
sageАноним 04/02/16 Чтв 16:27:40 #235 №335856 
14545924610800.jpg
>>335855
Аноним 04/02/16 Чтв 16:43:51 #236 №335857 
>>335849
Но 98%, с теми мощностями.
Аноним 04/02/16 Чтв 17:17:21 #237 №335863 
>>335856
Раскусил чертяка! когда в покер уже будем играть?
Аноним 04/02/16 Чтв 18:02:21 #238 №335875 
14545981414410.jpg
14545981414431.jpg
>>335184 (OP)
Матаны-батруханы
поясните пожалуйста мне дауну за
- мнимую единицу
- комплексные числа
- кватернионы
оче сильно хочу понять, не посылайте нахуй аргументируя что мне не дано...оче сильно жи хочется(((жи есть жи!!!!
вы моя последняя надежда
Аноним 04/02/16 Чтв 18:15:59 #239 №335877 
>>335875
Вообще, есть тред про элементарную математику, без философии и шизиков.
Что именно тебе непонятно?
Аноним 04/02/16 Чтв 18:22:09 #240 №335879 
>>335877
- нахуя вообще эту единицу придумали?
есть жи 0 на который делить нельзя и никто не выёбывается
- нахуя вообще комплексное число нужно, я так понел это тупо вектор с координатами?
- кватернион это вообще пздц, векторный поворот легко описывается матрицей координат, какие они блять там противоречия блять нашли вообще нихуя не понятно, и нахуй в кватернионе ввели понятие 4-го измерения, хватило бы трёх составляющих.
Аноним 04/02/16 Чтв 18:22:41 #241 №335880 
>>335877
и где ктати этот тред?
у вас тут не указателей ни ссылочек, вообще ничего нет.
Аноним 04/02/16 Чтв 18:39:42 #242 №335883 
>>335879
Это не тупо вектор с координатами, на комплексных числах можно ввести коммутативное ассоциативное умножение с делением, по теореме Фробениуса это единственный с точностью до изоморфизма такой объект над действительными числами.
Для чего они нужны - см. комплексный анализ и далее комплексную алгебраическую геометрию.
Аноним 04/02/16 Чтв 18:56:48 #243 №335886 
>>335879
Банальное объяснение про i:
В 16 веке хотели решать уравнения 3 и 4 степени, и для вывода потребовалось воспользоваться "трюком" - взятием корня из -1. Ввели i и -i.

В том же духе про кватернионы:
При помощи кватернионов удобно было записывать преобразования пространства.(Повороты, симметрии, перемещения, ...) Физикам понравилось, много кому ещё понравилось. Причём придумали кватернионы одновременно, если не раньше матриц.
Аноним 04/02/16 Чтв 18:59:39 #244 №335888 
>>335879
> есть жи 0 на который делить нельзя и никто не выёбывается
На ноль делить нельзя потому что если попытаться расширить математику так чтобы разрешить деление на ноль то получится хуита http://math.stackexchange.com/questions/125186/why-not-to-extend-the-set-of-natural-numbers-to-make-it-closed-under-division-by
А мнимую еденицу ввели - и получился удобный инструмент. Потому она и есть.
Аноним 04/02/16 Чтв 19:08:04 #245 №335889 
>>335700
>Её носитель не может быть даже собственным классом.
Почему не может? Разве на собственные классы накладываются какие-то ограничения?
Аноним 04/02/16 Чтв 19:30:09 #246 №335892 
>>335889
>Разве на собственные классы накладываются какие-то ограничения?
Да. Их элементами могут быть только множества, а не классы.
Аноним 04/02/16 Чтв 21:15:30 #247 №335897 
14546097302860.png
>>335892
Ну проблема-то решаема, если потребуется такая необходимость.
Аноним 04/02/16 Чтв 21:15:34 #248 №335898 
Вы случайно не знаете чувака, который весь предыдущий год создавал треде по математике в /b?
Как он поживает?
Аноним 04/02/16 Чтв 22:14:59 #249 №335907 
>>335888
>ссылка
спасибо почитал просветился
как будто корень из -1 это не ломание устоев и логики действий в алгебре.
Аноним 04/02/16 Чтв 22:21:48 #250 №335908 
>>335334
Потому что множеств в математике каждый день возникает слишком дохуя разных, чтобы учить всех только множеству полиномов или множеству рациональных чисел.
Аноним 04/02/16 Чтв 22:26:23 #251 №335909 
14546139836110.jpg
Репущу с гд:
Я пишу игру, сильно упростив ее можно свести к такой модели: на следующем шаге можно встретить врага, или восстановить здоровье. При встрече врага есть вероятность умереть. Как должны соотносится эти три параметра (вереятность умереть при встрече, вероятность встречи и вероятность регенерации) чтобы игра максимально затягивала?
Должна же быть теория какая-нибудь?
Аноним 04/02/16 Чтв 22:28:23 #252 №335910 
>>335184 (OP)
Можно ли доказать связность матричных групп типа GL(n), SL(n) и SO(n) геометрически, исходя из того, что задаются они неразложимым полиномом?
Аноним 04/02/16 Чтв 22:59:28 #253 №335913 
Есть прямая, на ней точка А и точка В, расстояние между точками скажем 2см. Могут ли существовать точки С и D на этой прямой, которые удалены от А на расстояние равное 1/3 см. и на расстояние 2^1/2 см. соответственно?

Если С и D существуют, то они существуют благодаря А и В или благодаря прямой?
Аноним 04/02/16 Чтв 23:38:07 #254 №335917 
>>335909
Тебе стоит сначала определить что такое "затягивала", в математическом смысле. Вообще для анализа подобных моделей используют цепи маркова.
Аноним 05/02/16 Птн 00:10:16 #255 №335923 
>>335913
Они существуют, потому что условия задачи не запрещают им существовать. Это же очевидно.
Аноним 05/02/16 Птн 00:45:21 #256 №335925 
>>335913
Прямая – неограниченное множество точек.
Аноним 05/02/16 Птн 02:08:37 #257 №335934 
Задачка школьная, на степени, но мозг спать хочет и не думает.
Есть карта со 100 000 персонажами. У каждого 4 характеристики с 4мя значениями каждая.
Какова вероятность встречи двух персонажей, у которых 4 из 4 характеристик имеют одинаковые значения? 3 из 4х? 3 из 4х, но трех персов?
Аноним 05/02/16 Птн 02:50:45 #258 №335937 
>>335934
Вероятность совпадения значений k из 4-х характеристик у n любых персонажей: p = C(4, k)(1/4)nk
Число вариантов выбора n персонажей из 100,000: C(100,000, n).
Требуемая вероятность: C(100,000, n)pn(1-p)100,000-n
Аноним 05/02/16 Птн 04:23:24 #259 №335944 
>>335770
Большое спасибо, анон.
Аноним 05/02/16 Птн 07:10:13 #260 №335960 
>>335898
До апреля скучать будет.
Аноним 05/02/16 Птн 09:28:17 #261 №335977 
Матаны такой вопрос. Достаточно ли только натуральных чисел для описания современных общепринятых физических теорий?
Аноним 05/02/16 Птн 09:29:22 #262 №335978 
>>335977
Толсто.
Аноним 05/02/16 Птн 09:38:53 #263 №335979 
>>335978
Блядь уже и вопрос нельзя задать. Сразу в толстоте обвиняют. Вы тут уже в конец доабстрагировались.
Аноним 05/02/16 Птн 09:43:52 #264 №335980 
>>335979
Расшифруй вопрос, что конкретно ты имеешь в виду. Как для тебя выглядит ситуация нехватки чисел в физической теории?
Аноним 05/02/16 Птн 09:49:12 #265 №335981 
>>335980
Наоборот, нету нехватки. Я подумал, что для описания стандартной модели и квантовой механики достаточно только натуральных чисел. Все остальные виды чисел в этой вселенной НЕ НУЖНЫ.
Аноним 05/02/16 Птн 09:56:42 #266 №335983 
>>335981
Нахуя ты сам с собой тут диалоги ведёшь, долбоёб?
Аноним 05/02/16 Птн 11:31:53 #267 №335991 
>>335981
Постоянная планка - это по-твоему натуральное число?
sageАноним 05/02/16 Птн 11:44:53 #268 №335995 
>>335981
Ты тот же поехавший, который Универсальный Сочинитель Теорем изобрёл или вы тут всё время новые?
Аноним 05/02/16 Птн 11:48:30 #269 №335997 
>>335991
Нужно все величины в физике умножить на постоянную Планка и тогда она станет натуральным числом.
Аноним 05/02/16 Птн 11:50:27 #270 №335998 
>>335991
Принимаем постоянную планка за единицу и пересчитываем все остальные величины исходя из этого. Природа этой вселенной квантовая, т.е. дискретная по сути. Все основанно на неделимых величинах, натуральных числах.
Аноним 05/02/16 Птн 11:56:12 #271 №335999 
>>335998
В любом случае тебе надо будет предполагать, что ты работаешь с непрерывными величинами, чтобы использовать мат аппарат, а потом каждый раз округлять, это будет тупо неудобно.
Аноним 05/02/16 Птн 12:04:07 #272 №336000 
>>335999
>В любом случае тебе надо будет предполагать, что ты работаешь с непрерывными величинами
Вот это непонятно. Объясни почему?
sageАноним 05/02/16 Птн 12:10:29 #273 №336001 
>>336000
>Вот это непонятно. Объясни почему?
Производную в школе уже прошли?
Аноним 05/02/16 Птн 12:19:45 #274 №336004 
>>335977
Кам минимум в атоме водорода есть один на эн квадрат, в осциляторе одна вторая, так что нельзя, вопрос закрыт.
sageАноним 05/02/16 Птн 12:46:18 #275 №336013 
>>336004
>>в атоме водорода есть один на эн квадрат
Нутак надо просто перевернуть дробь, это же условность

>>в осциляторе одна вторая
Надо домножить на два и всё
Аноним 05/02/16 Птн 12:51:13 #276 №336015 
>>336013
Это юмор такой? Я немного не понимаю.
Аноним 05/02/16 Птн 13:00:36 #277 №336021 
>>336015
Ты очень тупой
Аноним 05/02/16 Птн 14:28:18 #278 №336030 
Привет, я множество натуральных чисел. Меня зовут Кирилл.
Аноним 05/02/16 Птн 14:31:36 #279 №336031 DELETED
>>336030
Привет, я тупой школьник и буду тебя аксиоматизировать. Имя нам - Легион.
Аноним 05/02/16 Птн 14:32:26 #280 №336032 
>>335937
Спасибо, теперь вот буду вероятности вспоминать.
Аноним 05/02/16 Птн 14:40:23 #281 №336035 
>>336031
Я не люблю, чтобы меня кто-то аксиоматизировал. Я люблю дергать письку.
Аноним 05/02/16 Птн 15:22:57 #282 №336044 
Теория деформаций в алгебре крутая штука, да?
Аноним 05/02/16 Птн 15:42:45 #283 №336046 
>>336044
Нет, хуйня отстойная.
Аноним 05/02/16 Птн 15:52:32 #284 №336047 
>>336046
Почему?
Аноним 05/02/16 Птн 16:20:47 #285 №336055 
>>336035
Дерганье пиписьки состоит из следующих аксиом:
А Прислушиваться к шагам родителей
Б Тянуть вверх
В Тянуть вниз
Г Воображать всякое
Д Малафить

Любая комбинация этих строк есть фап. Если комбинация содержит Д, фап назвается завершенным.

Докажем, что твоя жизнь есть фап. Твой вчерашний день прелставляет собой:

АБВГАБГВАГБВГААААААААААГГБВАГГБВД
Аноним 05/02/16 Птн 16:31:06 #286 №336057 
Где модератор, блядь?!
Аноним 05/02/16 Птн 17:04:33 #287 №336061 
>>336057
Репорти блядь. Он не обязан за тобой бегать с тряпкой и подтирать говно.
Аноним 05/02/16 Птн 22:56:30 #288 №336126 
Как по-английски "надстойка" ?
Аноним 05/02/16 Птн 22:56:47 #289 №336127 
>>336126
* "надстройка"
sageАноним 05/02/16 Птн 23:03:19 #290 №336129 
>>336126
В /fl/.
Аноним 05/02/16 Птн 23:17:34 #291 №336131 
>>336127
add-ons
Аноним 05/02/16 Птн 23:34:43 #292 №336135 
>>336131
Нет, это же переводится как "добавить-включено"
Аноним 06/02/16 Суб 00:24:54 #293 №336138 
>>336135
add-on - в браузерах надстройка
penthouse - доп надстройка на зданиях
bells and whistles - доп свистелки и перделки

конкретное предложение дай, из какой сферы наук
Аноним 06/02/16 Суб 00:26:30 #294 №336139 
>>336138
https://ru.wikipedia.org/wiki/%D0%9D%D0%B0%D0%B4%D1%81%D1%82%D1%80%D0%BE%D0%B9%D0%BA%D0%B0_%28%D1%82%D0%BE%D0%BF%D0%BE%D0%BB%D0%BE%D0%B3%D0%B8%D1%8F%29 же
Аноним 06/02/16 Суб 00:29:37 #295 №336140 
>>336139
Suspension (topology)

в будущем просто кликай на ENGLISH в крайнем левом меню
sageАноним 06/02/16 Суб 00:33:07 #296 №336141 
>>336138
Съеби отсюда, лингвист хуев.

>>336139
Сука, ну как можно быть таким уебаном.
Аноним 06/02/16 Суб 00:34:09 #297 №336142 
>>336140
Спасибо, я не обращал внимания никогда на это меню.
>>336141
Чего горим?
Аноним 06/02/16 Суб 00:41:26 #298 №336144 
>>336141
Большая часть знаний созданных человеками переведена на английский или написана на нем. Когда будешь публиковать свои статьи на arxiv.org я помогу тебе с переводом.

P.S. за соответствующее вознаграждение of course.
Аноним 06/02/16 Суб 01:03:32 #299 №336147 
14547098122210.jpg
>>335913
существуют ли нерациональные углы?
Аноним 06/02/16 Суб 01:51:43 #300 №336149 
>>336147
Да.
Аноним 06/02/16 Суб 17:32:33 #301 №336220 
Как определить псевдовектор с помощью алгебр Грассмана?
Аноним 06/02/16 Суб 19:31:26 #302 №336227 
Посоветуйте пожалуйста годный задачник по математическому анализу, но такой, чтобы был с ответами, ибо надо проверять себя.
Аноним 06/02/16 Суб 19:48:28 #303 №336229 
>>336227
Очевидный Демидович + Антидемидович (антидемидовичи есть китайский и русский за авторством боярчука).
Аноним 06/02/16 Суб 20:26:30 #304 №336234 
>>336227
https://www.google.ru/search?q=%D0%9F%D0%BE%D0%BB%D0%B5+%D0%A1%D0%B5%D0%B3%D0%B5&rlz=1C1AVNE_enRU614RU614&oq=%D0%9F%D0%BE%D0%BB%D0%B5+%D0%A1%D0%B5%D0%B3%D0%B5&aqs=chrome..69i57&sourceid=chrome&es_sm=93&ie=UTF-8#newwindow=1&q=%D0%BF%D0%BE%D0%BB%D0%B8%D0%B0+%D1%81%D0%B5%D0%B3%D0%B5+%D0%B7%D0%B0%D0%B4%D0%B0%D1%87%D0%B8+%D0%B8+%D1%82%D0%B5%D0%BE%D1%80%D0%B5%D0%BC%D1%8B+%D0%B8%D0%B7+%D0%B0%D0%BD%D0%B0%D0%BB%D0%B8%D0%B7%D0%B0
Аноним 06/02/16 Суб 21:08:35 #305 №336238 
>>336229
> антидемидович
> китайский
?
Аноним 06/02/16 Суб 23:03:09 #306 №336248 
Как гуглить математическую наебку, где мы шар как-то делим на два шара равного объема?
Аноним 06/02/16 Суб 23:06:15 #307 №336249 
>>336248
Парадокс Банаха-Тарского
Аноним 07/02/16 Вск 00:16:10 #308 №336265 
>>336248
Только не объёма, а площади. И не на два шара, а на пять частей, из которых составляются два объекта. И не просто равные площади, а равные площади начального шара.
Аноним 07/02/16 Вск 01:15:56 #309 №336267 
>>336249
>Парадокс Банаха-Тарского
Теорема Банаха-Тарского
Аноним 07/02/16 Вск 01:17:41 #310 №336268 
>>336267
Сорь, не бейте
Аноним 07/02/16 Вск 01:41:19 #311 №336270 
>>335184 (OP)
> Малоизвестный нюанс здесь в том, что мы, вообще-то, не доказали, что совокупность {i1, i2, i3, ... } является множеством.

Не. В этом доказательстве другая дыра. Оно доказывает, что для любого N бесконечном множестве содержится не менее N элементов. Далее подразумевают, что это якобы доказывает, что в бесконечном множестве содержится счётное. Об этом нужно говорить аккуратнее.
Аноним 07/02/16 Вск 03:32:34 #312 №336272 
Я - кусок дерьма. Моя жизнь - кусок дерьма. Всё, что меня окружает - просто человеческий спам. Как и я сам.

Сдохнуть бы. Поскорее. Я заебался плакать по ночам.
Аноним 07/02/16 Вск 05:45:01 #313 №336281 
14548131012170.jpg
>>336220
Это вообще возможно? Я правильно понимаю суть тензоров?
Аноним 07/02/16 Вск 07:50:12 #314 №336296 
>>336272
Могу предложить утешающую задачку.
Аноним 07/02/16 Вск 08:03:51 #315 №336300 DELETED
>>336272
Понимаю тебя, бро. Это наваждение, бесы, они проверяют стойкость твоей души. Твоей дорогой смогут пройти немногие, и не жалуйся, что тебе живётся хуже других. Это твой путь.
Аноним 07/02/16 Вск 14:47:21 #316 №336406 
Некоторые точки, лежащие на данной алгебраической кривой лежат на другой алгебраической кривой заданной степени. Как можно найти эту кривую, куда копать?
Аноним 07/02/16 Вск 14:50:45 #317 №336409 
>>336406
Степень 1, ес-но
Аноним 07/02/16 Вск 14:51:17 #318 №336410 
>>336406
f(x) = g(x)^n
Аноним 07/02/16 Вск 15:04:05 #319 №336418 
>>336406
Это >>336409 не я, интересует в принципе любой степени.
>>336410
Первая кривая задана явно f(x) = 0.
У второй кривой задана степень n.
n не обязательно превосходит степень f.
Аноним 07/02/16 Вск 15:09:30 #320 №336420 
>>336406
Тащемта, если степень не задана четко, можно до бесконечности перечилять все варианты кривых.
Аноним 07/02/16 Вск 15:20:43 #321 №336426 
Вопросы от биомусора.
1. Метрика на функциях в R d0 не зависит от выбора координат - так(площадь между кривыми не меняется от поворотов-перемещений)? А почему это так?
2. Допустим, решение ОДУ методом разделения переменных - можно сделать без нотации dy/dt - и как обосновать перетасовывания дифференциалов по разные стороны равенства? Где-то встречал мнение, что эта нотация ущербна(правда не очень понял, осознавая её удобства). Подозреваю юзается инвариантность формы первого дифференциала(нам не важно что y - функция от t, а не наоборот, к примеру).
3. В интеграле фигурирует выражение f(x)dx. А что если вместо dx взять величину g(dx) -> 0 при dx -> 0 к примеру g(x)=x^k, e^x-1 и.т.д. и рассмотреть суммы, аналогичные интегральным - будут ли они существовать, сводиться к интегралам или у них своя собственная атмосфера?
4. Ну и встречал тут вопросец - формула производной обратной функции легальна или нет для частной производной, с пояснениями. Видимо опять же связь с инвариатностью дифференциала - ведь "частные" дифференциалы даже обозначаются по-разному(дельта малое vs латинского d).
Аноним 07/02/16 Вск 15:24:20 #322 №336428 
>>336418
Не про решение диффура ли ты говоришь?
Аноним 07/02/16 Вск 15:32:32 #323 №336430 
>>336426
1. Площадь определяется с помощью интеграла. Интеграл специально определён так, чтобы конгруэнтные фигуры (совмещающиеся движением: параллельным переносом, поворотом, симметрией) имели равную площадь.
Аноним 07/02/16 Вск 15:34:43 #324 №336432 
>>336430
Неочевидным мне кажется то, что определение интегральных сумм завязано на конкретные оси, по которым мы делаем разбиения.
Аноним 07/02/16 Вск 15:35:55 #325 №336433 
>>336432
Нарисуй ось в которой будет твой икс и игрек, если не ПОНИмаешь по другому.
Аноним 07/02/16 Вск 15:40:31 #326 №336440 
>>336426
Ну есть же тред про элементарную математику!

>В интеграле фигурирует выражение f(x)dx. А что если вместо dx взять величину g(dx) -> 0 при dx -> 0

При стремлении аргумента dx к нулю g(dx) превратится, в сущности, просто в линейную функцию от dx. Я полагаю что это будет эквивалентно растягиванию или сжатию оригинального интеграла по оси х.

Например, пусть g'(0)=5, в таком случае g(dx)=5dx. Итого имеем формулу:

Int f(x)g(dx)= int f(x)g'(0) dx = g'(0) int f(x) dx
Аноним 07/02/16 Вск 15:46:59 #327 №336442 
>>336426
>формула производной обратной функции легальна или нет для частной производной

Легальна. После фиксации всех переменных кроме одной получаем, в сущности, функцию одной переменной, для которой справедливо y'=1/x'.

>>336440
Аноним 07/02/16 Вск 16:25:59 #328 №336477 
>>336428
Нет, а какой дифур?
Аноним 07/02/16 Вск 16:33:18 #329 №336479 
>>336477
Я тупанул, никакой это не диффур
Аноним 07/02/16 Вск 20:37:42 #330 №336576 
Какой алгебраической поебенью можно описать ньютоновскую физику? Комбинаторы, клеточные автоматы? Или не ньютоновскую, а какую-нибудь выдуманную.

ссылки на работы с таким - было бы хорошо.
Аноним 07/02/16 Вск 20:50:37 #331 №336578 
>>336576
Ньютоновская механика - тривиальный случай. Луркай гамильтонову механику как структуру на симплектическом многообразии и "Математические методы классической механики" Арнольда.
Аноним 07/02/16 Вск 21:27:38 #332 №336582 
Из статьи "Топологическое пространство" https://ru.wikipedia.org/wiki/%D0%A2%D0%BE%D0%BF%D0%BE%D0%BB%D0%BE%D0%B3%D0%B8%D1%87%D0%B5%D1%81%D0%BA%D0%BE%D0%B5_%D0%BF%D1%80%D0%BE%D1%81%D1%82%D1%80%D0%B0%D0%BD%D1%81%D1%82%D0%B2%D0%BE

Пусть дано множество X. Система T его подмножеств называется тополо́гией на X, если выполнены следующие условия:

1 Объединение произвольного семейства множеств, принадлежащих T, принадлежит T
2 Пересечение конечного семейства множеств, принадлежащих T, принадлежит T
3 Пустое множество принадлежит T

Бля, да в каких случаях пункты 1,2 не будут выполняться? По-моему, они будут выполняться всегда. Это же очевидно из определения Кантором множества.

И уточнение о конечном семействе во 2 пункте зачем?
Аноним 07/02/16 Вск 21:29:41 #333 №336584 
>>336582
Возможно тебя смутило словосочетание "система подмножеств". Имеется в виду, что T --- подмножество P(X), где P(X) --- множество всех подмножеств X.
Аноним 07/02/16 Вск 21:48:32 #334 №336586 
>>336584
Черт. Может есть какая-то занимательная топология для дебилов?
Аноним 07/02/16 Вск 21:52:40 #335 №336587 
>>336584
семейство множеств тождественно системе множеств?
Аноним 07/02/16 Вск 21:53:22 #336 №336589 
>>336587
>тождественно
Или в случае определений лучше эквивалентно?
Аноним 07/02/16 Вск 22:14:17 #337 №336592 
>>336586
Кстати, есть. Гугли наглядную топологию.
Аноним 08/02/16 Пнд 00:17:17 #338 №336595 
>>336281
Что такое псевдовектор? Почитал википедию, ничего не понял, вектор, не меняющий направление относительно отражений чего блядь несёшь.
Аноним 08/02/16 Пнд 06:23:15 #339 №336617 
>>336595
Я не знаю и пытаюсь понять. Это термин из физики.

>>336582
>да в каких случаях пункты 1,2 не будут выполняться?
Пусть M = {a,b,c,d} - четырёхточечное множество. Пусть T = { {}, {a,b,c,d}, {a,b}, {b,c} }. Объединение {a,b} и {b,c} есть {a,b,c} - не элемент T. Пересечение {a,b} и {b,c} есть {b} - не элемент T.
>И уточнение о конечном семействе во 2 пункте зачем?
Бесконечное пересечение открытых множеств может не быть открыто.
Например, рассмотрим вещественную прямую R, откроем в ней все интервалы (a,b), a<b, и всевозможные их бесконечные объединения и конечные пересечения. В такой топологии - это известный факт - отрезки не будут открыты.
Рассмотрим следующее бесконечное семейство открытых множеств. {(-1; 2), (-0.5; 1.5), (-0.25; 1.25), (-0.125; 1.125), ... } Пересечение этого семейства есть отрезок [0;1]. Он не открыт.
Аноним 08/02/16 Пнд 09:14:45 #340 №336637 
Посоветуйте англоязычный форум о матане.
Аноним 08/02/16 Пнд 09:30:57 #341 №336638 
>>336637
4chan.org/sci
Аноним 08/02/16 Пнд 09:36:25 #342 №336640 
>>336637
>матан
>ан
Ты, блядина, серьёзно?
Аноним 08/02/16 Пнд 09:58:45 #343 №336648 
>>336638
Хотелось бы именно форум, а не борду и не реддит.

>>336640
Чем тебе этот самолёт не угодил?
Аноним 08/02/16 Пнд 10:09:09 #344 №336652 
>>336648
http://mathoverflow.net сойдёт?
Аноним 08/02/16 Пнд 10:14:50 #345 №336655 
>>336648
Потому что ты хочешь форум по математике, а не матанализу, долбоёб.
Аноним 08/02/16 Пнд 10:28:21 #346 №336660 
>>336652
Нет, меня интересует именно форум.
Аноним 08/02/16 Пнд 10:48:23 #347 №336661 
>>336617
Спасибо
Аноним 08/02/16 Пнд 11:38:08 #348 №336667 
>>336576
Клеточные автоматы это уже нихуя не ньютон.
Аноним 08/02/16 Пнд 12:01:22 #349 №336671 
>>336667
В клеточном автомате выразим не только Ньютон, но и Энштейн. Клеточные автоматы в общем случае Тьюринг-полные.
Аноним 08/02/16 Пнд 12:01:49 #350 №336672 
>>336576
Тензоры.
Аноним 08/02/16 Пнд 13:09:36 #351 №336709 
>>336671
Законы логики это не физика.
Аноним 08/02/16 Пнд 13:24:15 #352 №336711 DELETED
>>336709
Да ну?! Гениус.жпг.
Аноним 08/02/16 Пнд 14:02:17 #353 №336727 
>>336711
Ну смотри, клуточные автоматы работают на логике.
У физики тоже есть логика, только она своя собственная. Например, есть у нас электрическое поле. Почему вместе с ним вознивает магнитное? Ведь они никак не связаны. На этот вопрос ответить не можем пока, так как не понимаем логики по которой они работают.
Аноним 08/02/16 Пнд 14:03:49 #354 №336730 
А есть форум как /sci/ только без ёбанных даунов? Очень нужно.
Аноним 08/02/16 Пнд 14:09:41 #355 №336735 
>>336730
dxdy
Аноним 08/02/16 Пнд 14:26:13 #356 №336740 DELETED
>>336727
Еблан, естественно речь не идёт о том, что клеточный автомат является физикой (что бы этот твой тезис ни значил). Тебе говорят о возможности в правилах клеточного автомата репрезентовать любые физические законы. Это и называют полнотой по Тьюрингу.
Аноним 08/02/16 Пнд 14:31:30 #357 №336746 
>>336740
Мамашу свою репрезентируй.
Аноним 08/02/16 Пнд 15:15:37 #358 №336773 
>>336727
Пиздос бред
Аноним 08/02/16 Пнд 15:16:51 #359 №336775 
>>336727
>У физики тоже есть логика, только она своя собственная.
Чот чувствую себя, будто в религиозную секту попал.
Аноним 08/02/16 Пнд 15:32:37 #360 №336784 
>>336740
Репрезентуй мне квантовую неопределенность.
Аноним 08/02/16 Пнд 15:38:55 #361 №336789 
>>336784
if, else
Аноним 08/02/16 Пнд 16:21:57 #362 №336797 
>>336576
Арнольда тебе уже вкинули, а всякое говно типа
> Комбинаторы, клеточные автоматы
оставь при себе
Аноним 08/02/16 Пнд 16:23:39 #363 №336798 
>>336789
Ясно.
Пройдите нахуй уважаемый.
Аноним 08/02/16 Пнд 16:36:19 #364 №336801 
>>336797
но ведь у арнольда матан, а не алгебра.
Аноним 08/02/16 Пнд 16:38:31 #365 №336803 
>>336798
даун думает, что кв неопределенность нельзя формализовать какой-нибудь тривиальной функцией.
Аноним 08/02/16 Пнд 16:39:14 #366 №336804 
>>336801
А теперь быстро, решительно поясни за разницу.
Аноним 08/02/16 Пнд 17:02:49 #367 №336806 
>>336804
>не видеть разницу между математическим анализом и алгеброй
Или ты из тех пэтэушников, кто математику матаном называет?
Аноним 08/02/16 Пнд 17:06:21 #368 №336807 
>>336798
Очередной тупорылый воняющий водкой тупорылый физик лезет пиздеть за математику
Аноним 08/02/16 Пнд 17:13:32 #369 №336809 
>>336807
Я специально два раза написал, что ты тупорылый, чтобы до тебя дошло
Аноним 08/02/16 Пнд 17:15:40 #370 №336810 
>>336806
Мне действительно интересно, как ты отличаешь матан от алгебры. Скажем, теорема о том, что многочлен ненулевой степени с коэффициентами из C имеет по крайней мере один корень в C, относится к алгебре или к матану? Ведь все её доказательства используют топологию.
Аноним 08/02/16 Пнд 17:15:55 #371 №336811 
>>336801
а тогда в сарданашвили загляни, тебе понравится
Аноним 08/02/16 Пнд 17:34:41 #372 №336813 
>>336810
А́лгебра (от араб. اَلْجَبْرْ‎‎, «аль-джабр» — восполнение[1]) — раздел математики, который можно грубо охарактеризовать как обобщение и расширение арифметики.
Математи́ческий ана́лиз (классический математический анализ) — совокупность разделов математики, соответствующих историческому разделу под наименованием «анализ бесконечно малых», объединяет дифференциальное[⇨] и интегральное[⇨] исчисления.
На классическом математическом анализе основывается современный анализ, который рассматривается как одно из трёх основных направлений математики (наряду с алгеброй и геометрией)
Аноним 08/02/16 Пнд 17:35:12 #373 №336814 
>>336813
лол
Аноним 08/02/16 Пнд 17:46:56 #374 №336815 
>>336813
Этот текст не даёт способа отличить анализ от алгебры. Попробуй ещё раз.
Аноним 08/02/16 Пнд 17:53:21 #375 №336817 
>>336810
К алгебре.
Аноним 08/02/16 Пнд 17:58:27 #376 №336820 
>>336817
К анализу.
Аноним 08/02/16 Пнд 17:59:20 #377 №336822 
>>336820
К алгебре.
Аноним 08/02/16 Пнд 17:59:36 #378 №336823 
>>336822
К анализу.
Аноним 08/02/16 Пнд 18:00:49 #379 №336824 
>>336823
А ты умеешь настоять на своем. Го ебаться.
Аноним 08/02/16 Пнд 18:01:32 #380 №336825 
>>336824
Го.
Аноним 08/02/16 Пнд 18:06:14 #381 №336827 
>>336825
Напиши мне. Мой имэйл [email protected]
Аноним 08/02/16 Пнд 18:11:33 #382 №336829 
>>336827
Такой имейл не зарегистрирован. Я проверил.
Ты обманщик.
Аноним 08/02/16 Пнд 18:14:36 #383 №336830 
>>336829
Прости, если я задел твои чувства. Это не потому, что я хотел тебя обидеть, а потому, что я не хочу ебаться с мужиком.
knuebok 08/02/16 Пнд 20:01:38 #384 №336848 
>>336810
Не все, есть и чисто алгебраические доказательства.
Аноним 08/02/16 Пнд 20:02:24 #385 №336849 
>>336803
Даун настолько не в теме, что даже не знает, что кв неопределенность - это не функция, лол.
Аноним 08/02/16 Пнд 20:33:39 #386 №336857 
>>336849
ну а чем она формализуется?
Аноним 08/02/16 Пнд 20:52:47 #387 №336862 
>>336857
в операторах
Аноним 08/02/16 Пнд 20:55:34 #388 №336864 
>>336862
Разве нельзя все свести к множествам и отображениям, как основе матемики?
Аноним 08/02/16 Пнд 20:58:24 #389 №336865 
>>336864
Т.е что такое оператор ты не знаешь? В таком случае этот диалог бессмысленен.
Аноним 08/02/16 Пнд 20:58:33 #390 №336866 
14549543133000.jpg
Как вы относитесь к профессору Фоменко и его методу распутывания истории?
Аноним 08/02/16 Пнд 21:02:12 #391 №336868 
>>336866
Фоменко - математик это норм.
Фоменко - историк это треш.
Аноним 08/02/16 Пнд 21:02:49 #392 №336869 
>>336865
>конечный автомат, надроченный на мехмате, не нашел подходящего паттерна.
Ок
Аноним 08/02/16 Пнд 21:03:26 #393 №336870 
>>336869
провернул автомат в твоем анусе
Аноним 08/02/16 Пнд 21:11:03 #394 №336873 
>>336809
>нихуя не понял о чем разговор
>развонялся на всю нить
Твое мнение очень важно для меня. Пожалуйста, формализируй псевдовекторы по бурбаки.
Аноним 08/02/16 Пнд 21:13:20 #395 №336874 
14549552002000.jpg
>>336864
>оператор
>свести к множествам и отображениям
У меня есть одна охуительная картинка на этот случай.
А вот и она.
Аноним 08/02/16 Пнд 21:15:49 #396 №336875 
14549553494520.jpg
>>336220
Бамп вопросу.
Аноним 08/02/16 Пнд 21:47:07 #397 №336880 
14549572278100.jpg
>>336875
Если нужно что-то подобное, то смотри во 2 томе Лорана Шварца
Аноним 08/02/16 Пнд 21:52:10 #398 №336881 
>>336880
>Лорана Шварца
Внезапно. Хорошо, спасибо.
Аноним 08/02/16 Пнд 21:54:42 #399 №336882 
Пустое множество является элементом множества?
Аноним 08/02/16 Пнд 21:58:41 #400 №336884 
>>336882
Смотря какого. Пустое множество является элементом множества {∅, 1, 2, 3}, но не является элементом множества {1,2,3}. ∅ не является элементом ∅.

Тем не менее, для любого множества X верно, что ∅⊂X. Ибо не нужно путать значки ⊂ и ∈.
Аноним 08/02/16 Пнд 22:25:58 #401 №336891 
Я задал этот вопрос, потому что не понимаю, как задать множество всех подмножеств.
Известно, что количество элементов в множестве всех подмножеств в некоем множестве А равняется 2 в степени n, где n — количество элементов этого самого множества А.

Возьмём множество А = { {a, b}, a, b}. В нём содержатся, как я понимаю, три элемента — {a, b}, a и b.
Тогда, следуя вышеизложенному правилу, количество элементов в множестве P всех подмножеств множества А равно 2^3 = 8.
Но пытаясь задать это самое множество Р, мы получаем ({a, b} — один элемент): Р = {∅, {a, b}, a, b, {{a, b}, a}, {{a, b}, b}, {{a, b}, a, b}} — всего семь элементов, хотя должно быть, как мы видели, восемь! Почему не хватает одного элемента? А потому что по комбинаторным правилам этим элементом должно являться множество второго и третьего элементов исходного множества: {a, b}, но у нас оно уже есть, потому что содержится в изначальном множестве А, а элементы множества, как известно, не могут повторяться!
Внимание, вопрос: так, блядь, где я ошибся? Ведь согласно формуле должно быть восемь элементов, а из-за первоначального набора приходится избегать повтора и выходит на один меньше! Почему? Что я не понял?

Ебучая, блядь, теория множеств. Абсолютно неудобная.
Аноним 08/02/16 Пнд 22:26:15 #402 №336892 
>>336891
>>336884
Аноним 08/02/16 Пнд 22:29:42 #403 №336893 
Есть таблично заданное множество S и функция f(x_1, ..., x_n) -> R, x_i ∈ S. Как найти min(f)?
|S| = 1000, n = 50
Точное решение не нужно.
Мне кажется что-то подходящие должно быть в discrete optimization, но мне пока не удалось ничего найти. Может кто-нибудь что-нибудь подсказать?
И что бы понятнее было: S - множество строк и его первые элементы могут выглядеть так: {alice, bob, cat, dog, abu, 2ch, ...}
Аноним 08/02/16 Пнд 22:51:17 #404 №336905 
>>336891
>так, блядь, где я ошибся?
Ты забыл ещё одно одноэлементное множество: {{a, b}}.
Аноним 08/02/16 Пнд 22:54:31 #405 №336906 
>>336905
Оно эквивалентно {{a, b}, ∅}?
Аноним 08/02/16 Пнд 22:58:02 #406 №336907 
14549614827780.jpg
>>336906
Нет. Ты написал множество их двух элементов: {a, b} и пустого множества.
Аноним 08/02/16 Пнд 22:58:46 #407 №336908 
>>336891
>Р = {∅, {a, b}, a, b, {{a, b}, a}, {{a, b}, b}, {{a, b}, a, b}}
Неверно.
Аноним 08/02/16 Пнд 22:59:56 #408 №336910 
>>336907
Спасибо! Всё понятно.
Аноним 08/02/16 Пнд 23:17:50 #409 №336917 
Группа на n+1 образующих $s_i$, с соотношениями, очень похожими на перестановки
$s_i^2 = 1$, $s_i s_j = s_j s_i$ для $|i-j| > 1$ кроме $i=0,j=n$, $s_i s_{i+1} s_i = s_{i+1} s_i s_{i+1}$, но только ещё и $s_0$ и $s_n$ в таком же соотношении, как будто они рядом $s_0 s_n s_0 = s_n s_0 s_n$. Как доказать что эта группа изоморфна $S_{n+1}$ полупрямо на решётку $Z^n$ ?
Аноним 08/02/16 Пнд 23:46:39 #410 №336922 
Скиньте кто-нибудь N-петуха.
Аноним 08/02/16 Пнд 23:55:28 #411 №336923 
Ибо ваистену, множество спасет мир! Гаварим мир - подразумеваемЪ множество, гаварим множесто - падразумиваим мир.
Аноним 09/02/16 Втр 00:11:45 #412 №336924 
>>336917
Проиграл с этого пэхэпэшника.
Аноним 09/02/16 Втр 00:20:12 #413 №336926 
>>336924
Мне даже лень комментировать.
sageАноним 09/02/16 Втр 01:19:52 #414 №336947 
>>336922
Откуда?
Аноним 09/02/16 Втр 01:22:58 #415 №336949 
Блядь, а ведь уже 2016.
Аноним 09/02/16 Втр 01:39:28 #416 №336959 
>>336949
А ты ещё ничего не доказал и не создал, уныло, да?
Даже Мочизуки прочитать не в состояние, что ты делал всё это время? Пил ягуар в 8 классе вместо того, чтобы изучать топологию? Выпились
Аноним 09/02/16 Втр 01:49:00 #417 №336967 
14549717404400.jpg
>>336922
Прошу
Аноним 09/02/16 Втр 01:49:53 #418 №336968 
>>336959
Нет, зачем выпиливаться? Я хочу хотя бы Галуа пережить.
Аноним 09/02/16 Втр 02:25:23 #419 №336983 
Похуй. Нихуя я не могу.
Аноним 09/02/16 Втр 03:57:52 #420 №336995 
>>336968
Он мёртв, например, так что выпиливайся
Аноним 09/02/16 Втр 05:48:20 #421 №337001 
Для доказательства каких более-менее простых утверждений используются р-адические числа?
Аноним 09/02/16 Втр 14:25:10 #422 №337037 
Господа, объясните на понятном языке что такое тензор
с меня два пива
Аноним 09/02/16 Втр 15:12:25 #423 №337047 
>>337037
Многомерная матрица.
Аноним 09/02/16 Втр 16:08:03 #424 №337061 
>>337037
Тензор над пространством V - это вектор из тензорного произведения нескольких экземпляров пространства V и нескольких экземпляров пространства Vd, двойственного к V.

Тензорное произведение векторных пространств A1, A2, ... , An - это векторное пространство V, снабжённое отображением g: A1×A2×...×An→V и обладающее следующим фундаментальным свойством.

Обозначим символом A декартово произведение A1×A2×...×An.
Пусть E - любое векторное пространство, и пусть f - любое полилинейное отображение из A в E.
Тогда существует единственное отображение h:V→E такое, что для любого x из A верно, что f(x) = h(g(x)).
Аноним 09/02/16 Втр 16:35:42 #425 №337068 
>>337037
Это универсальное полилинейное отображение. Совсем грубо говоря — к обычным векторам добавлена операция крест в кружке (типа умножение, это и есть «отображение»), можно раскрывать скобки в линейных комбинациях («полилинейность»). «Унивесальность» означает что больше ничего делать нельзя.
sageАноним 09/02/16 Втр 18:31:06 #426 №337088 
>>336891
Да ты картофан какой-то, там всё через когомологии видно, дебил.
Аноним 09/02/16 Втр 18:55:49 #427 №337096 
14550333493170.jpg
>>337088
Вы только посмотрите, кое-кто пытается в перефорс.
Аноним 09/02/16 Втр 19:30:05 #428 №337107 
14550354053450.png
Кабздос, неужели я додвачевался до того что забыл элементарную алгебру? Поясните как там оно все вычислилось? Хочу понимать как оно там устроенно.
Аноним 09/02/16 Втр 21:24:42 #429 №337139 
>>337107
Хрень какая-то.
Аноним 09/02/16 Втр 23:17:06 #430 №337158 
Что изучает топология?
Аноним 09/02/16 Втр 23:17:27 #431 №337159 
>>337107
Легко.
Аноним 09/02/16 Втр 23:18:33 #432 №337161 
>>337158
понятие непрерывности в широком смысле.
Аноним 09/02/16 Втр 23:19:11 #433 №337162 
>>337161
А что изучает непрерывность?
Аноним 09/02/16 Втр 23:21:36 #434 №337163 
>>337162
Непрерывность - это концепция, а не учёный, она ничего изучать не может.
Аноним 09/02/16 Втр 23:22:11 #435 №337164 
>>337163
А что изучает эта концепция?
knuebok 09/02/16 Втр 23:24:42 #436 №337166 
>>337164
Концепции, обычно, ничего не изучают. По-поводу топологии можешь почитать хороший ответ тут: http://math.stackexchange.com/questions/60152/motivation-behind-topology
Аноним 09/02/16 Втр 23:29:29 #437 №337167 
Посоветуйте самую абстрактную область математики
Аноним 09/02/16 Втр 23:29:48 #438 №337168 
>>337166
А что они тогда изучают?
knuebok 09/02/16 Втр 23:30:50 #439 №337169 
>>337168
Ничего не изучают - они отражают некоторые явления или взаимосвязи в реальном мире или в манямирке математика. Или не отражают.
Аноним 09/02/16 Втр 23:32:39 #440 №337171 
>>337167
>>337169
И чтобы было оче концептуально сложно, максимум абстрактности, лучше даже, если таких объектов не было или они были очень очень сложно находимы и доказуемы
Примерно как сейчас теория струн для начала физики
Аноним 09/02/16 Втр 23:32:44 #441 №337172 
>>337169
А что изучают явления?
Аноним 09/02/16 Втр 23:36:08 #442 №337173 
>>337171
Ну сейчас идёт микрохайп по поводу интеруниверсальной теории Мочидзуки Тейхмюллера, не знаю, насколько это соответствует твоим требованиям, у меня лвла не хватает понять даже обзоры на неё.

>>337172
Природные? Физики, наверное.
Аноним 09/02/16 Втр 23:39:52 #443 №337175 
>>337173
С чего начинать подъём к горе Мочидзуки-Тейхмюллера?
Аноним 09/02/16 Втр 23:41:58 #444 №337177 
>>337175
А у тебя какой уже левел?
Аноним 09/02/16 Втр 23:42:23 #445 №337178 
>>337175
Со всего, лол.
Аноним 09/02/16 Втр 23:42:56 #446 №337179 
>>337173
Что изучают явления?
Аноним 09/02/16 Втр 23:51:30 #447 №337184 
>>337166
На всякий случай задам вопрос про псевдовектор ещё и персонально тебе, няша. >>336220
Ссылку на Лорана Шварца я уже получил, но ещё не осилил.
Аноним 09/02/16 Втр 23:55:05 #448 №337190 
>>337184
Что изучают явления?
Аноним 10/02/16 Срд 00:03:25 #449 №337191 
>>337177
второй судя по пику
knuebok 10/02/16 Срд 00:07:27 #450 №337193 
>>337184
http://math.stackexchange.com/questions/264771/cross-product-and-pseudovector-confusion хорошее объяснение.
Аноним 10/02/16 Срд 00:10:48 #451 №337196 
>>337193
Привет. Что изучают явления?
Аноним 10/02/16 Срд 00:25:43 #452 №337202 
>>337193
Спасибо. Хотя конструкция всё ещё выглядит неестественной, но суть я, похоже, понял.
Аноним 10/02/16 Срд 01:10:59 #453 №337203 
>>337171
Для чего интересуешься?
Аноним 10/02/16 Срд 01:33:44 #454 №337208 
>>337203
Для чего интересуешься?
Аноним 10/02/16 Срд 02:22:03 #455 №337224 
>>337175
С матшколы в детстве, кружков и хождения в нму с 15 лет. Да, ты опоздал.
Аноним 10/02/16 Срд 02:26:03 #456 №337225 
>>337224
Но мне 14
Аноним 10/02/16 Срд 08:38:52 #457 №337253 
Чем может быть механика с точки зрения математики?
Аноним 10/02/16 Срд 09:58:25 #458 №337256 DELETED
>>337253
Например?
Аноним 10/02/16 Срд 10:09:08 #459 №337258 
>>336893
бамп
Аноним 10/02/16 Срд 10:21:00 #460 №337259 
>>337256
Что изучают явления?
Аноним 10/02/16 Срд 10:56:10 #461 №337263 
>>337256
Ну тогда общий способ синтеза механик.
Аноним 10/02/16 Срд 11:16:14 #462 №337266 
>>337159
Ну поясни тогда как.
Аноним 10/02/16 Срд 11:59:21 #463 №337273 
Что изучают явления?
Аноним 10/02/16 Срд 12:21:38 #464 №337277 DELETED
>>337273
НЛО
Аноним 10/02/16 Срд 12:22:12 #465 №337278 
>>337259
Это Что изучают явления? не я писал. Мне интересны абстрактные механики. И синтез и/или определение механики в рамках математики.
Аноним 10/02/16 Срд 12:24:24 #466 №337280 
>>336874
Хули все так эклектично? Вот, например, есть язык, из него можно составить любую поебень, а в математике не так? Суки, где базис и основания?
Аноним 10/02/16 Срд 12:29:13 #467 №337282 
14550965538250.jpg
14550965538281.jpg
Как относитесь к творчеству Вавилова? Стоит ли обмазатся его курсом https://www.lektorium.tv/course/26552 ? Хочу как можно быстрее вкатится в современную алгебру, понимать необходимый минимуму для чтения более-менее современных статей.
Аноним 10/02/16 Срд 12:41:49 #468 №337285 
Существуют ли обобщения анализа для произвольных отношений (в противовес функциям)? Могут ли они иметь смысл?
Аноним 10/02/16 Срд 13:01:51 #469 №337290 DELETED
>>337285
https://ru.m.wikipedia.org/wiki/%D0%A0%D0%B5%D0%BB%D1%8F%D1%86%D0%B8%D0%BE%D0%BD%D0%BD%D0%B0%D1%8F_%D0%B0%D0%BB%D0%B3%D0%B5%D0%B1%D1%80%D0%B0
Аноним 10/02/16 Срд 13:04:40 #470 №337291 
>>337290
> Существуют ли обобщения анализа
> анализа
> алгебра

Нихуево обобщил, ничего не скажешь.
Аноним 10/02/16 Срд 13:25:21 #471 №337297 
Советую всем здешним переключится на ДУЛКОЛАКС, все прояснится и образуется. Я серьезно!
Аноним 10/02/16 Срд 13:28:04 #472 №337300 
>>337282
Вавилов умный, но ебанулся. Впрочем, его ёбнутость пока ещё не зашла далеко и не вредит работоспособности. Просто фильтруй все лишнее.
Аноним 10/02/16 Срд 13:28:07 #473 №337301 
>>337285
А что такого особенного в отношениях, что матан не должен их изучать? Например, отношение на R геометрически представляет собой какую-то область на плоскости. Естественно, это изучается в матане.
Аноним 10/02/16 Срд 14:05:06 #474 №337304 
14551023070210.png
Посоны, у меня получилось, что интеграл любой простой функции равен половине квадрата этой функции.
Аноним 10/02/16 Срд 14:29:22 #475 №337307 
Посоны, закончил вузик 3 года назад. Естественно все забыл к хуям.
И вот тут ВНЕЗАПНО захотелось понять в Фурье-образы, ибо через них работает быстрое сравнение изображений, а через сабж - поиск фрагментов в компьютерном зрении.

Посоветуйте источников, чтобы прокачать матан с нуля? Читал википедию и охуевал - каждое второе слово непонятное. Причем в вузике этого преобразования Фурье то и не было нихуя, были ряды, а это совсем другая ебань.
Аноним 10/02/16 Срд 14:32:55 #476 №337308 
14551039753040.jpg
>>337304
Это как?
Аноним 10/02/16 Срд 15:01:18 #477 №337318 
>>337307
>матан с нуля
https://www.coursera.org/learn/calculus1
Аноним 10/02/16 Срд 15:04:46 #478 №337321 
>>337308
Magic.
Аноним 10/02/16 Срд 15:05:38 #479 №337323 
>>337307
>Читал википедию и охуевал - каждое второе слово непонятное.
Например?
Аноним 10/02/16 Срд 15:08:51 #480 №337324 
>>337321
Ты произвольную функцию как t представил, у тебя уже нет права считать t рандомной переменной, следовательно t' в данном случае уже не 1, а... f'(x).
Аноним 10/02/16 Срд 15:09:35 #481 №337325 
>>337323
Линейный оператор, свертка, равенство Парсеваля, гильбертово пространство, функция хевисайда, дельта Дирака, пространство Шварца.

Блять, кто все эти люди?
Аноним 10/02/16 Срд 15:14:14 #482 №337328 
>>337323
Например "педия". Что это вообще значит?
Аноним 10/02/16 Срд 15:16:03 #483 №337330 
>>337328
Моя собака любит Педигри.
Аноним 10/02/16 Срд 15:25:28 #484 №337334 
>>337325
на кого и где учился?
Аноним 10/02/16 Срд 15:29:34 #485 №337338 
>>337307
Рудин. Только упражнения не пытайся делать, они там пиздец сложные. Просто просматривай все упражнения и делай самые простые и самые важные.
sageАноним 10/02/16 Срд 15:57:43 #486 №337354 
>>337328
От "энциклопедия".
Аноним 10/02/16 Срд 16:05:43 #487 №337362 
>>337334
Да какая нафиг разница.
Аноним 10/02/16 Срд 17:33:54 #488 №337381 
>>337362
интересно просто, что это за место где об линейных операторах не знают.
Аноним 10/02/16 Срд 17:40:35 #489 №337382 
>>337107
Ну бамп
Аноним 10/02/16 Срд 17:59:56 #490 №337386 
>>337382
Легко там всё.
Аноним 10/02/16 Срд 18:17:12 #491 №337388 
>>337382
Сначала раскрываешь скобки, а потом сворачиваешь по теореме Христоса-Шварца.
Аноним 10/02/16 Срд 18:37:03 #492 №337390 
>>337282
Курс по ссылке не смотрел, но имеющиеся в сети книги за его авторством (конкретная теория колец и конкретная теория групп) содержат кучу отсебятины, минимальное количество общеполезного материала и вообще отдают фриковатостью. Подозреваю, что курс состоит из того же самого.
Современная алгебра - это алгебра Ленга (современное издание) или любой аналог, Атья Макдональд или Matsumura по коммутативной алгебре и Weibel по гомологической алгебре. Для "чтения более-менее современных статей" этого недостаточно, но знать это необходимо, т.к. эти сведения считаются общеизвестными.
Аноним 10/02/16 Срд 18:46:35 #493 №337394 
>>337390
>имеющиеся в сети книги за его авторством (конкретная теория колец и конкретная теория групп)
Гораздо интереснее поговорить о его теории множеств, http://rghost.ru/6XGdThvR4
Можешь ли ты подвергнуть критике эту книгу?
Аноним 10/02/16 Срд 18:46:51 #494 №337395 
>>337390
Есть серия годных лекций по абстрактной алгебры?
Аноним 10/02/16 Срд 18:47:16 #495 №337396 
>>337394
Вирус некачайтие
Аноним 10/02/16 Срд 18:48:23 #496 №337398 
>>337396
Само собой.
Аноним 10/02/16 Срд 19:06:03 #497 №337399 
>>337282
Лично мне очень нравится.
>>337390
>содержат кучу отсебятины
Да.
>минимальное количество общеполезного материала
Нет. Материала у него наоборот больше, просто излагается неспешно.
>и вообще отдают фриковатостью
Ничего подобного, если, конечно,
фриковатостью не называть всё, что немного отличается.
>Подозреваю, что курс состоит из того же самого.
Неправильно подозреваешь, вообще записанные лекции у него заметно «стандартнее».
Аноним 10/02/16 Срд 19:18:20 #498 №337402 
>>337390
Посмотрел первую лекцию на ускорении 2x. За первые сорок минут он рассказал историю алгебры, напал на христианство, продемонстрировал русофобию и англоманию, прорекламировал теорию категорий и гомологическую алгебру, рассказал про своих крутых предков, а также высмеял специалистов по дифференциальным уравнениям с их страстью писать триста статей про решение одного дифура.

Затянуло. Смотрю дальше.
Аноним 10/02/16 Срд 19:18:41 #499 №337403 
>>337399
Вавилов плес
Аноним 10/02/16 Срд 19:18:59 #500 №337404 
>>337394
Не буду и пытаться, т.к. вообще не разбираюсь в этой теме
>>337390
Они весьма многочисленны - есть видеолекции матфака ВШЭ и НМУ, плюс куча курсов от MIT, Гарварда и т.д.
На самом деле, почти все содержат стандартный материал:группы, кольца, поля, линейная алгебра, теория Галуа, с включением чего-то из коммутативной алгебры или алгебр Ли на усмотрение автора, так что особой разницы что смотреть по-моему нет
Аноним 10/02/16 Срд 19:28:24 #501 №337405 
>>337399
Под "минимальным количеством общеполезного материала" я подразумевал не полную бесполезность, а лишь сомнительность а качестве базового учебника. Простые идеалы и спектр кольца в последней главе - слишком странное решение чтобы быть оправданным.
Аноним 10/02/16 Срд 19:29:26 #502 №337406 
>>337405
>в качестве
Аноним 10/02/16 Срд 19:29:28 #503 №337407 
Математики, есть проблема. Есть нелинейная электрическая цепь из 3х последовательно включённых элементов с известными ВАХ. Все 3 вах монотонно неубывающие. Конкретнее: тунельный зазор, активное сопротивление и диод Шоттки. Нужно найти ВАХ этой цепи в виде аналитической функции. Нужно для подбора параметров к точкам, полученным в результате эксперимента.
Аноним 10/02/16 Срд 19:30:37 #504 №337408 
>>337404
Только что досмотрел до содержания курса Вавилова, как его видит Вавилов. В содержание входят теория представлений, полилинейная алгебра (ну, поливекторы), теория категорий и (sic!) гомологическая алгебра.

>>337405
Это не учебники, это объекты искусства, о чём он ещё во введении заявил. Разве тебя не заинтересовал факт, что почти все эпиграфы в книжке выдуманы лично Вавиловым? Об этом он сам предупреждает.
Аноним 10/02/16 Срд 19:32:02 #505 №337409 
Как я понимаю, "Algebra 1" читают на первом курсе, а "Algebra 2" - на втором?
Аноним 10/02/16 Срд 19:35:48 #506 №337410 
>>337409
А Algebra 3 - на третьем
Аноним 10/02/16 Срд 19:38:43 #507 №337411 
>>337405
>Под "минимальным количеством общеполезного материала" я подразумевал не полную бесполезность, а лишь сомнительность а качестве базового учебника.
Пожалуй, согласен. Слишком много отвлекается.
>Простые идеалы и спектр кольца в последней главе - слишком странное решение чтобы быть оправданным.
То, что лежит в интернете по теории колец — адово сырое. Это черновики всё, выложенные не автором.
Вполне доработанная первая часть теории групп: http://ir.nmu.org.ua/bitstream/handle/123456789/19792/db5229cc07d857754e34fe4a61b6cbda.pdf?sequence=1
намного законченнее демонстрирует стиль сабжа.
Аноним 10/02/16 Срд 19:42:21 #508 №337412 
>>337408
Ну ОК, "объект искусства" так "объект искусства", на вкус и цвет все фломастеры разные. Я также готов признать способность тов. Вавилова написать вводный курс алгебры для второкурсников - вопрос закрыт.
>>337411
а где тогда искать полную версию, есть ли она в природе?
Аноним 10/02/16 Срд 19:43:48 #509 №337413 
>>337412
>а где тогда искать полную версию, есть ли она в природе?
Неизвестно.
Аноним 10/02/16 Срд 20:02:06 #510 №337414 
>>336893
Скажите хотя бы в какую сторону копать?
Аноним 10/02/16 Срд 20:16:23 #511 №337419 
>>336893
Никак, блеать, перебором, если про функцию ничего не известно и там в принципе значения в каждой точке могут быть любыми, никак не зависящими друг от друга — какие тут могут быть методы. И что значит «точное решение не нужно», вообще пушка.
Аноним 10/02/16 Срд 20:58:40 #512 №337442 
>>337419
> если про функцию ничего не известно и там в принципе значения в каждой точке могут быть любыми
Там не всё рандомно, закономерности есть. Но нужен метод, который сам бы понял эти закономерности.
> что значит «точное решение не нужно», вообще пушка.
Может я не так выразился. Мне не обязательно нужен глобальный минимум, достаточно близкого к нему значения.
Аноним 10/02/16 Срд 21:27:37 #513 №337452 
>>337414
Local search.
Аноним 10/02/16 Срд 21:35:35 #514 №337456 
Поясните, не понимаю какой алгоритм (или группа алгоритмов) для вычисления группы галуа? Вот есть у нас соотношения, которые автоморфизмы поля должны сохранять, тогда понятно что предъявление таких уравнений отсекает лишние перестановки, не являющиеся автоморфизмами. Допустим, мы попредъявляли достаточно и нашли некоторое множество перестановок, а больше никаких соотношений, не следующих из уже рассмотренных у нас не получается. Как доказать что их и вправду нет?
Аноним 10/02/16 Срд 21:38:33 #515 №337459 
>>336893
Интересная задача.

Как насчет исследовать функцию методом градиентного спуска, получить набор описывающих минимумы векторов, затем скомпоновать на основе твоей таблицы похожие вектора и, подставив, сравнить значения?

Аноним 10/02/16 Срд 21:47:10 #516 №337463 
>>337459
>>337442
>>336893
один хер придется перебрать весь массив, чтобы знать что есть минимум. а когда перебрал уже и знаешь какой минимум.
Аноним 10/02/16 Срд 21:48:53 #517 №337464 
>>337452
да,годная тема
Аноним 10/02/16 Срд 22:24:48 #518 №337474 
>>337459
Что-то я не представляю себе как тут можно градиент рассчитать.
Пусть S = {a, b, ..., z}, n = 1. Как мне рассчитать grad(f) в точке m?

>>337452
На первый взгляд это очень похоже на брутфорс, но я посмотри завтра подробнее. Спасибо!
Аноним 10/02/16 Срд 23:03:41 #519 №337481 
>>337474
так это оно и есть, только подбираются алгоритмы подбора, а не тупо варианты. типа первая производная брутфорса по методам лол
Аноним 10/02/16 Срд 23:06:50 #520 №337483 
>>337474
ну он предлагает сперва взять и прогнать вообще все возможные значения через эту функцию. и уже заранее знать что в точке м должен быть градиент. после чего, когда соберешся сортировать конкретный массив, предлагает подставить эти, заранее известные значения. и проверить оно канает на самом деле или нет.
Аноним 10/02/16 Срд 23:35:22 #521 №337485 
>>337463
> Один хер придётся перебрать все точки, в которых функция на R может принимать значения.
Аноним 10/02/16 Срд 23:44:49 #522 №337486 
Ебать, чё либген лежит так долго.
sageАноним 11/02/16 Чтв 03:25:16 #523 №337527 
>>336891
Первак, марш с двача нахуй! Ща Хана позову, он твою life сделает miserable.
Аноним 11/02/16 Чтв 04:05:17 #524 №337533 
>>337304
А у меня получилось, что производная любой дифференцируемой функции равна 1.
f'(x)=[сделаем замену t=f(x)]=t'=1
Аноним 11/02/16 Чтв 09:37:21 #525 №337565 
>>335184 (OP)
Пагни, есть ли смысл вкатываться в математику (для себя), с бездуховной литературы?
https://sites.google.com/site/scienceandmathguide/subjects/mathematics

Годны ли эти книжонки, читал ли кто? Или не выёбуваться, и читать на русском?
Аноним 11/02/16 Чтв 09:53:04 #526 №337566 
>>337409
Нет, необязательно. Главное, что каждый курс занимает 1 семестр и проходить курсы надо именно в таком порядке.
Аноним 11/02/16 Чтв 10:00:47 #527 №337567 
>>337565
> есть ли смысл вкатываться в математику (для себя)
нет

> Годны ли эти книжонки, читал ли кто? Или не выёбуваться, и читать на русском?
Рудин норм, остальные хуйня.
Аноним 11/02/16 Чтв 10:07:11 #528 №337568 
>>337567
>нет
Ты ответил на вопрос, который не был задан
Аноним 11/02/16 Чтв 10:28:57 #529 №337576 
>>337568
Я просто сделал более сильное утверждение. Из того, что нет смысла вкатываться в математику для себя, следует, что нет смысла вкатываться в математику для себя по любой литературе.
Аноним 11/02/16 Чтв 10:57:22 #530 №337583 
зачем в комплексном числе записывают мнимые числа и реальные числа через знак "+" ? мне кажется было бы логичнее просто записывать пары чисел через запятую, не было бы лишней путаницы, всё равно никаких операций сложения и вычитания между реальными числами и мнимыми быть не может.
Аноним 11/02/16 Чтв 11:09:39 #531 №337585 
>>337583
> сложения и вычитания между реальными числами и мнимыми быть не может.
Когда мы работаем в C, под вещественными числами мы имеем в виду не вещественные числа, а подполе C, изоморфное полю вещественных чисел, так что все может.
Перекатs Аноним 11/02/16 Чтв 11:37:14 #532 №337595 
>>337592 (OP)
>>337592 (OP)
>>337592 (OP)
Аноним 12/02/16 Птн 00:07:15 #533 №337915 
14552248353270.webm
Немножко шабата в тред.
Аноним 15/02/16 Пнд 01:24:54 #534 №338955 
3
comments powered by Disqus

Отзывы и предложения